Vous êtes sur la page 1sur 101

PROBLEMAS PREPARATORIOS

CON

RESPUESTAS
Melbourne, Australia, 5 14 de julio de 1998

Traducidos por Ivn Tubert

30a Olimpiada Internacional de Qumica. Problemas preparatorios

Contenido
Nota a los mentores.........................................................................................iii Jurado Cientfico de la 30a Olimpiada Internacional de Qumica..............v Temario provisional para la Olimpiada Internacional de Qumica...........vi Problemas..........................................................................................................1
problema 1 1 PROBLEMA 2...............................................................................................................................2 problema 3 4 problema 4 4 problema 5 5 problema 6 6 problema 7 7 problema 8 7 problema 9 8 problema 10 10 problema 11 10 problema 12 11 problema 13 12 Problema 14 13 problema 15 15 problema 16 16 problema 17 17 problema 18 18 Problema 19 18 Problema 20 19 Problema 21 20 problema 22 21 Problema 23 23 Problema 24 23 Problema 25 24 Problema 26 25 Problema 27 - Experimental.......................................................................................................28 Problema 28 - Experimental.......................................................................................................29 Problema 29 - Experimental.......................................................................................................32 Problema 30 - Experimental.......................................................................................................34

Respuestas a los problemas...........................................................................38


Problema 1 38 Problema 2 39 problema 3 41 Problema 4 43 problema 5 44

Melbourne, Australia. Julio de 1998

30a Olimpiada Internacional de Qumica. Problemas preparatorios


Problema 6 45 Problema 7 46 Problema 8 46 Problema 9 47 Problema 10 49 Problema 11 51 Problema 12 52 Problema 13 54 Problema 14 55 Problema 15 57 Problema 16 60 Problema 17 62 Problema 18 65 Problema 19 66 Problema 20 68 Problema 21 71 Problema 22 72 Problema 23 74 Problema 24 77 Problema 25 79 Problema 26 81

ndice Alfabtico.............................................................................................84

Melbourne, Australia. Julio de 1998

ii

30a Olimpiada Internacional de Qumica. Problemas preparatorios

Nota a los mentores


Durante la 29a Olimpiada Internacional de Qumica en Montreal, el jurado internacional decidi reducir el nmero de problemas preparatorios tericos de alrededor de 50 (lo cual haba sido la costumbre en los ltimos aos) a 25. Hemos alcanzado este objetivo (casi tenemos 26 problemas tericos, pero como vern posteriormente, el PROBLEMA 26 es ms bien una gua de estudio) y nos hemos esforzado para que los ejercicios cubran todos los temas de nivel 3 que sern representados en los exmenes de la 30 a IChO en Melbourne. Para su conveniencia, hemos incluido la lista de los temas que son generalmente aceptados como base para las preguntas de examen en las Olimpiadas Internacionales de Qumica. Este es el mismo temario que recibimos de Montreal. Los problemas han sido diseados para retar y estimular los intereses de los mejores estudiantes de Qumica a nivel preuniversitario de su pas. Tambin hemos incluido respuestas detalladas para cada problema y esperamos que sus estudiantes aprendan bastante Qumica de estas respuestas detalladas sin que se necesite la supervisin continua de los mentores. Este ao tenemos cuatro ejercicios detallados para el laboratorio que cubren las habilidades que deben poseer los estudiantes que vayan a Melbourne. En cada ejercicio hemos tratado de recalcar los procedimientos que requieren de especial cuidado, aun para estudiantes de nivel olmpico, pero nuestras advertencias no son exhaustivas los estudiantes necesitarn de la supervisin de los mentores. Tampoco incluimos detalles especficos sobre el manejo y desecho de los productos de estos ejercicios de laboratorio, ya que varan de pas en pas, pero sabemos ustedes se encargarn de desechar o reciclar de manera responsable los materiales que sus estudiantes utilicen o produzcan. Ciertamente, los estudiantes debern estar conscientes de los riesgos asociados con los productos qumicos que se utilicen en cada ejercicio, por lo que les sugerimos que les informen adecuadamente. A nuestros jvenes colegas que pasarn muchas horas los prximos meses pensando acerca de estos ejercicios, les daremos la oportunidad de discutir estos ejercicios (y otros asuntos) con sus compaeros estudiantes de todo el mundo, aun antes de que vengan todos a Melbourne. Hemos preparado un foro de chat en el WWW para que puedan conocerse (despus de todo, no es este uno de los principales objetivos de las Olimpiadas?). Les sugerimos que informen a sus estudiantes acerca de este foro, y si as lo desean, entren a la discusin ustedes mismos. Finalmente, a pesar de los valientes esfuerzos de revisin de mis colegas, sera sorprendente que no encontraran algn error o dos por ah. Estas deficiencias son por supuesto mi responsabilidad* , as que no duden en avisarme. Regularmente se enviarn correcciones por correo electrnico a cada delegacin, y se publicarn en la pgina de la red cuya direccin se muestra a continuacin. Tambin estarn disponibles las copias corregidas de este documento en varios formatos en esta misma direccin. Buena suerte en su preparacin, y esperamos ansiosos para encontrarnos en Melbourne.
*

N. del T. Evidentemente tambin puede haber errores de traduccin. Para quejas y sugerencias, escrbanme a itubert@hotmail.com

Melbourne, Australia. Julio de 1998

iii

30a Olimpiada Internacional de Qumica. Problemas preparatorios

Alan Arnold Presidente del Jurado Cientfico de la 30a IChO apa@adfa.oz.au

La pgina de los problemas preparatorios en el WWW se encuentra en http://www.ch.adfa.oz.au/ASO/IChO/30IChO/PrepProblems.html

Melbourne, Australia. Julio de 1998

iv

30a Olimpiada Internacional de Qumica. Problemas preparatorios

Jurado Cientfico de la 30a Olimpiada Internacional de Qumica


Dr Patricia Angus Dr Alan Arnold (Chair) Assoc. Prof. Neil Barnett Dr Ross Coller Dr Susan Cumming Prof Dainis Dakternieks Mrs Carolyn Elvins Dr Greg Klease Dr Simon Petrie Prof Colin Raston Prof. Richard Russell Dr Greg Simpson Dr Suzanne Smith Dr Brian Yates Australian National University University College (UNSW) ADFA Deakin University University of Melbourne Royal Australian Chemical Institute Deakin University Presbyterian Ladies College Central Queensland University University College (UNSW) ADFA Monash University Deakin University CSIRO Molecular Science ANSTO University of Tasmania

Melbourne, Australia. Julio de 1998

30a Olimpiada Internacional de Qumica. Problemas preparatorios

Temario provisional para la Olimpiada Internacional de Qumica


Clasificacin de los temas Grupo 1: Estos temas se incluyen en la inmensa mayora de los programas de Qumica de nivel preuniversitario. Grupo 2: Estos temas no se incluyen en muchos de los programas de Qumica de nivel preuniversitario; sin embargo, se espera que los estudiantes de Qumica de nivel olmpico de cualquier pas hayan estudiado estos temas. Grupo 3: Estos temas no se incluyen en casi ningn programa de nivel preuniversitario. Ya no es necesario que el pas anfitrin elabore problemas preparatorios para los temas de los grupos 1 y 2, aunque, en este ltimo caso, se debe dar una lista de los temas especficos del grupo 2 que pueden ser utilizados en el examen. Todos los temas del grupo 3 que puedan aparecer en el examen de la Olimpiada debern estar cubiertos en los problemas preparatorios.

QUMICA INORGNICA
CONFIGURACIN ELECTRNICA
1 2 3 4 5 grupos principales 1 metales de transicin 2 metales lantnidos y actnidos 3 Principio de exclusin de Pauli 1 Regla de Hund 1

16

propiedades trmicas 3 estructuras metlicas 3 estructuras de cristales inicos. Estructuras moleculares simples con tomo central 3 estructuras que exceden la regla del octeto 3 estereoqumica 3 compuestos de los grupos principales 1 compuestos de los metales de transicin 1 complejos metlicos simples 2 complejos metlicos con ms de un centro 3 nmero de coordinacin 1 balanceo de ecuaciones 1 relaciones de masa y volumen 1 frmula emprica 1 nmero de Avogadro 1 clculos de concentraciones 1 conteo de nucleones 1

ESTRUCTURAS
17 18

19 20

TENDENCIAS EN LA TABLA PERIDICA (GRUPOS PRINCIPALES)


6 7 8 9 10 11 electronegatividad 1 afinidad electrnica 2 primera energa de ionizacin 2 tamao atmico 1 radio inico 2 mximo nmero de oxidacin 1

NOMENCLATURA
21 22 23 24 25

ESTEQUIOMETRA
26 27 28 29 30

TENDENCIAS EN LAS PROPIEDADES FSICAS (GRUPOS PRINCIPALES)


12 13 14 15 punto de fusin 1 punto de ebullicin 2 carcter metlico 1 propiedades magnticas 2

ISTOPOS
31

Melbourne, Australia. Julio de 1998

vi

30a Olimpiada Internacional de Qumica. Problemas preparatorios


32 33 decaimiento radiactivo 1 reacciones nucleares (alfa, beta, gama, neutrino) 2 54 55 otros tiocidos, policidos y peroxicidos 3 los estados de oxidacin ms comunes de los elementos del 2o y 3er perodos en compuestos con halgenos o en oxoaniones son: B(III), Al(III), Si(IV), P(V), S(IV), S(VI), O(-2), F(-1), Cl(I), Cl(III), Cl(V), Cl(VII). 1 compuestos de nometales en otros estados de oxidacin 3 los estados de oxidacin preferidos son Sn(II), Pb(II), Bi(III) 2 los productos de reaccin de los xidos no metlicos con el agua. Estequiometra de los cidos resultantes 1 reaccin de los halgenos con el agua 2 la reactividad y el poder oxidante de los halgenos decrece del F2 al I2 1 las diferencias entre la qumica de los elementos del 3er y 4o perodos 3

CICLOS NATURALES
34 35 36 nitrgeno 2 oxgeno 2 carbono 2 productos de la reaccin con agua. Basicidad de los productos 1 productos de la reaccin con halgenos 1 productos de la reaccin con oxgeno 2 los elementos ms pesados son ms reactivos 1 el Li se combina con H2 y N2, formando LiH y Li3N 2 58 56

BLOQUE s
37

57

38 39 40 41

59 60

61

BLOQUE p
42 estequiometra de los hidruros nometlicos simples 1 propiedades de los hidruros metlicos 3 propiedades cido/base de CH4, NH3, H2S, H2O, HX 1 reaccin del NO con O2 para formar NO2 1 equilibrio entre el NO2 y el N2O4 1 productos de reaccin del NO2 con el agua 1 el HNO2 y sus sales son reductores 1 el HNO3 y sus sales son oxidantes 1 el N2H4 es un lquido reductor 3 existen cidos como el H2N2O2, HN3 3 recordar cules son los productos de la reduccin de nitratos o HNO3 con diferentes metales o reductores 3 reaccin del Na2S2O3 con el yodo 2

BLOQUE d
62 los estados de oxidacin comunes de los metales ms comunes del bloque d son: Cr(III), Cr(VI), Mn(II), Mn(IV), Mn(VII), Fe(II), Fe(III), Co(II), Ni(II), Cu(I), Cu(II), Ag(I), Zn(II), Hg(I), Hg(II) 1 color de los iones anteriores en solucin acuosa 2 otros estados de oxidacin. Qumica de otros elementos del bloque d 3 el Cr, Mn, Fe, Ni, Co y Zn se disuelven en HCl diluido; el Cu, Ag y Hg no se disuelven 1 los productos de la disolucin son cationes (2+) 2 pasivacin de Cr, Fe (y Al) 2 el Cr(OH)3 y el Zn(OH)2 son anfteros, mientras que otros hidrxidos comunes no 1

43 44

45 46 47 48 49 50 51 52

63

64

65

66

67

53

Melbourne, Australia. Julio de 1998

vii

30a Olimpiada Internacional de Qumica. Problemas preparatorios


68 el MnO4-, CrO42- y Cr2O72- son oxidantes fuertes 1 productos de reduccin de MnO4- en funcin del pH 2 polianiones distintos del Cr2O723

ALQUINOS
90 91 geometra lineal 1 acidez 2

69

ARENOS
92 frmula del benceno 1 93 deslocalizacin de electrones 1 94 estabilizacin por resonancia 1 95 regla de Hckel (4n + 2) 3 96 aromaticidad de heterociclos 3 97 nomenclatura (IUPAC) de heterociclos 3 98 compuestos aromticos policclicos 3 efecto del primer sustituyente 99 en la reactividad 2 100 en la orientacin 2 101 explicacin de los efectos del sustituyente 2

70

OTROS PROBLEMAS INORGNICOS


71 produccin industrial de H2SO4, NH3, Na2CO3, Na, Cl2, NaOH 1 qumica de los lantnidos y actnidos 3 qumica de los gases nobles 3

72 73

QUMICA ORGNICA
ALCANOS
74 75 76 ismeros del butano 1 nomenclatura (IUPAC) 1 tendencias en propiedades fsicas 1 sustitucin (p. ej. con Cl2): productos 1 radicales libres 2 iniciacin y terminacin de la reaccin en cadena 2 80 81 82 cicloalcanos nomenclatura tensin en los anillos pequeos 2 conformacin de silla y de bote 2 planaridad 1 isomera E/Z (cis/trans) 1 adicin de Br2, HBR: productos 1 regla de Markovnikoff 2 carbocationes en la reaccin de adicin 3 estabilidad relativa de los carbocationes 3 adicin 1,4 a dienos 3

COMPUESTOS CON HALGENOS


102 reaccin de hidrlisis 2 103 intercambio de halgenos 3 104 reactividad (primario, secundario, terciario) 2 105 mecanismo inico 2 106 productos secundarios (eliminacin) 2 107 reactividad (aliftico vs aromtico) 2 108 reaccin de Wurtz (RX + Na) 3 109 derivados halogenados y contaminacin 3

77 78 79

ALQUENOS
83 84

ALCOHOLES Y FENOLES
110 puentes de hidrgeno (alcoholes vs teres) 1 111 acidez de alcoholes vs fenoles 2 112 deshidratacin a alquenos 1 113 deshidratacin a teres 2 114 steres con cidos inorgnicos 2 115 reaccin del yodoformo 2 116 reactividad de alcoholes primarios, secundarios y terciarios: reactivo de Lucas

85 86 87 88 89

Melbourne, Australia. Julio de 1998

viii

30a Olimpiada Internacional de Qumica. Problemas preparatorios


140 amidas a partir de cloruros de acilo 2 141 nitrilos a partir de cloruros de acilo 3 142 propiedades y preparacin de anhdridos 2 143 nombre y frmula del cido oxlico 1 144 cidos multifuncionales 2 145 actividad ptica (p. ej. cido lctico) 2 146 nomenclatura R/S 3 147 grasas animales vs vegetales diferencias 2

2 117 frmula de la glicerina 1

COMPUESTOS CARBONLICOS
118 nomenclatura 1 119 tautomera ceto/enol 2 preparacin 120 oxidacin de alcoholes 1 121 con monxido de carbono 3 reacciones 122 oxidacin de aldehdos 1 123 reduccin con Zn metlico 2 124 adicin de HCN 2 125 adicin de NaHSO3 2 126 adicin de NH2OH 2 127 condensacin aldlica 3 128 Canizzaro (desproporcionacin del PhCH2OH) 3 129 reaccin de Grignard 2 130 Fehling (Cu2O) y Tollens (espejo de Ag) 2

COMPUESTOS CON NITRGENO


148 basicidad de las aminas 1 149 diferencias entre aromticas y alifticas 2 150 nomenclatura: primarias, secundarias, terciarias, cuaternarias 2 151 identificacin de aminas primarias, secundarias, terciarias y cuaternarias en el laboratorio 3 preparacin de aminas 152 a partir de compuestos halogenados 2 153 a partir de nitrocompuestos (PhNH2 a partir de PhNO2) 3 154 a partir de amidas (Hoffman) 3 155 mecanismo de la reaccin de Hoffman en medio cido y bsico 3 basicidad de aminas vs amidas 2 productos de diazotacin de aminas alifticas 3 de aminas aromticas 3 colorantes: color en funcin de la estructura (grupos cromforos)

CIDOS CARBOXLICOS
131 efecto inductivo y fuerza cida 2 132 equivalencia de los dos tomos de oxgeno en los aniones 2 preparacin 133 a partir de steres 2 134 a partir de nitrilos 2 135 productos de reaccin con alcoholes (esterificacin) 1 mecanismo de la esterificacin 2 istopos en la elucidacin de mecanismos 3 nomenclatura de cloruros de acilo 2 preparacin de cloruros de acilo 2

156

157 158 159

136 137 138 139

3 160 nitrocompuestos: tautomera nitro/aci 3 161 rearreglos de Beckmann (oximaamida) 3

MACROMOLCULAS
162 grupos hidroflicos e hidrofbicos 2

Melbourne, Australia. Julio de 1998

ix

30a Olimpiada Internacional de Qumica. Problemas preparatorios


163 estructura de las micelas 3 164 preparacin de jabones 1 productos de polimerizacin de 165 estireno 2 166 eteno 1 167 poliamidas 3 168 fenol + aldehdos 3 169 poliuretanos 3 polmeros 170 entrecruzamiento 3 171 estructuras (isotctica, etc.) 3 172 mecanismo de formacin en cadena 2 173 composicin del hule 3 188 desnaturalizacin por cambio de pH, temperatura, metales, EtOH) 2 189 estructura cuaternaria 3 190 separacin de protenas (tamao molecular y solubilidad) 3 metabolismo de las protenas (general) 3 protelisis 3 transaminacin 3 cuatro rutas para el catabolismo de los aminocidos

191 192 193 194

3 195 descarboxilacin de aminocidos 3 196 ciclo de la urea (solamente los resultados) 3

GRASAS Y CIDOS GRASOS


197 nombres de la IUPAC del C4 al C18 2 198 nombres triviales de los ms importantes (aproximadamente cinco) 2 199 metabolismo general de las grasas 3 200 oxidacin de los cidos grasos (frmulas y balance de ATP) 3 anabolismo de grasas y cidos grasos 3 fosfoglicridos 3 membranas 3 transporte activo 3

BIOQUMICA
AMINOCIDOS Y PPTIDOS
174 estructura inica de los aminocidos 1 175 punto isoelctrico 2 176 clasificacin de los 20 aminocidos en grupos 2 177 estructuras de los 20 aminocidos 3 178 reaccin de la ninhidrina (incluyendo ecuacin) 3 179 separacin cromatogrfica 3 180 separacin por electroforesis 3 181 enlace peptdico 1

201 202 203 204

ENZIMAS
205 propiedades generales, centros activos 2 206 nomenclatura, cintica, coenzimas, funcin del ATP, etc. 3

PROTENAS
182 estructura primaria de las protenas 1 183 puentes de disulfuro 3 184 anlisis de secuencia 3 185 estructura secundaria 3 186 detalles sobre la estructura de hlice 3 187 estructura terciaria 3

CARBOHIDRATOS
glucosa y fructosa 207 frmula de cadena 2 208 proyeccin de Fischer 2 209 proyeccin de Haworth 3

Melbourne, Australia. Julio de 1998

30a Olimpiada Internacional de Qumica. Problemas preparatorios


210 osazonas 3 211 maltosa como azcar reductor 2 212 diferencia entre almidn y celulosa 2 213 diferencia entre alfa y betaD glucosa 2 214 metabolismo del almidn a la acetilCoA 3 ruta metablica hacia el cido lctico o etanol 215 catabolismo de la glucosa 3 216 balance de ATP 3 217 fotosntesis (nicamente productos) 2 218 reaccin en la luz y en la oscuridad 3 219 ciclo de Calvin detallado 3 235 DNA ligasa 3 236 sntesis de RNA (transcripcin) sin detalles 3 237 transcriptasa reversa 3 238 uso del cdigo gentico 3 239 codones de inicio y fin 3 240 pasos de la traduccin 3

OTRA BIOQUMICA
241 hormonas y regulacin 3 242 retroalimentacin hormonal 3 243 insulina, glucagon, adrenalina 3 244 metabolismo de minerales (sin detalles) 3 245 iones en la sangre 3 246 buffers en la sangre 3 hemoglobina 247 funcin y esqueleto 3 248 diagramas de absorcin de oxgeno 3 249 pasos en la coagulacin de la sangre 3 250 antgenos y anticuerpos 3 251 grupos sanguneos 3 252 estructura y funciones de la acetilcolina 3

CICLO DE KREBS Y RESPIRACIN


220 formacin de CO2 en el ciclo (sin detalles) 3 221 compuestos intermediarios en el ciclo 3 222 formacin de agua y ATP (sin detalles) 3 223 FMN y citocromos 3 224 clculo de la cantidad de ATP a partir de 1 mol de glucosa 3

SNTESIS DE PROTENAS Y CIDOS NUCLEICOS


225 pirimidina y purina 2 226 nuclesidos y nucletidos 3 227 frmulas de todas las bases de purina y pirimidina 3 228 diferencia entre la ribosa y la 2-desoxiribosa 3 229 combinacin de bases CG y AT 3 230 estructura de puentes de hidrgeno en la combinacin de CG y AT 3 diferencias entre el RNA y el DNA 3 diferencias entre mRNA y tRNA 3 hidrlisis de los cidos nucleicos 3 replicacin semiconservativa del DNA 3

MTODOS INSTRUMENTALES DE DETERMINACIN DE ESTRUCTURAS


ESPECTROSCOPA UVVISIBLE
253 identificacin de compuestos aromticos 3 254 identificacin del cromforo 3

ESPECTROMETRA DE MASAS
255 identificacin del ion molecular 3 256 identificacin de fragmentos con ayuda de una tabla 3 257 distribucin isotpica tpica 3

231 232 233 234

IR
258 interpretacin de espectros usando una tabla 3

Melbourne, Australia. Julio de 1998

xi

30a Olimpiada Internacional de Qumica. Problemas preparatorios


259 identificacin de puentes de hidrgeno 3 260 espectroscopa Raman 3

EQUILIBRIO INICO
280 teora de Arrhenius de cidos y bases 1 281 teora de Brnsted y Lowry; cidos y bases conjugados 1 282 definicin de pH 1 283 producto inico del agua 1 284 relacin entre Ka y Kb para cidos y bases conjugados 1 285 hidrlisis de sales 1 286 producto de solubilidad (definicin) 1 287 clculo de solubilidades en agua a partir del producto de solubilidad 1 288 clculo del pH para un cido dbil a partir de Ka 1 clculo del pH del HCl 10-7 M 2 clculo del pH de cidos poliprticos 2 definicin del coeficiente de actividad 2 definicin de la fuerza inica 3 ecuacin de DebyeHckel 3

RMN
261 interpretacin de espectros simples (como el del etanol) 3 262 acoplamiento spinspin 3 263 constantes de acoplamiento 3 264 identificacin del benceno disustituido orto y para 3 265 RMN de 13C 3

RAYOS X
266 ley de Bragg 3 267 diagramas de densidad electrnica 3 268 nmero de coordinacin 3 269 celda unitaria 3 estructuras tipo 270 NaCl 3 271 CsCl 3 272 empaquetamiento compacto (dos tipos) 3 273 clculo de la constante de Avogadro a partir de datos de rayos X 3

289 290 291 292 293

POLARIMETRA
274 clculo del ngulo de rotacin especfica 3

EQUILIBRIO DE ELECTRODOS
294 fuerza electromotriz (definicin) 1 295 electrodos del primer tipo 1 296 potencial estndar de electrodo 1 297 ecuacin de Nernst 2 298 electrodos del segundo tipo 2 299 relacin entre G y fuerza electromotriz 3

FISICOQUMICA
EQUILIBRIO QUMICO
275 modelo dinmico del equilibrio qumico 1 equilibrio expresado en trmino de 276 concentraciones relativas 1 277 presiones parciales relativas 2 278 relacin entre las diferentes constantes de equilibrio para gases ideales (concentraciones, presiones, fraccin mol) 2 279 relacin entre la constante de equilibrio y la energa libre de Gibbs estndar

CINTICA DE REACCIONES HOMOGNEAS


300 factores que afectan la velocidad de reaccin 1 301 ecuacin de velocidad 1 302 constante de velocidad 1

Melbourne, Australia. Julio de 1998

xii

30a Olimpiada Internacional de Qumica. Problemas preparatorios


303 orden de reaccin 2 reacciones de 1er orden 304 concentracin en funcin del tiempo 2 305 vida media 2 306 relacin entre vida media y constante de velocidad 2 307 paso determinante de la velocidad 2 308 molecularidad 2 309 ecuacin de Arrhenius. Energa de activacin (definicin) 2 310 clculo de la constante de velocidad para reacciones de 1er orden 2 311 clculo de la constante de velocidad para reacciones de 2o y 3er orden 3 312 clculo de la energa de activacin a partir de datos experimentales 3 313 conceptos bsicos de teora de colisiones 3 314 conceptos bsicos de teora del estado de transicin 3 315 reacciones opuestas, paralelas y consecutivas 3 326 energas de enlace (definicin y usos) 2

SEGUNDA LEY
327 definicin de entropa (q/T) 2 328 entropa y desorden 2 329 relacin S = k lnW 3 330 relacin G = H - TS 2 331 G y direccionalidad de los cambios 2

SISTEMAS DE FASES
332 ley del gas ideal 1 333 ley de gases de van der Waals 3 334 definicin de presin parcial 1 335 dependencia de la presin de vapor de un lquido con respecto a la temperatura 2 ecuacin de Clausius-Clapeyron 3 interpretacin de diagramas de fases punto triple 3 temperatura crtica 3 sistemas lquidovapor diagrama 3 sistemas ideales y no ideales 3 usos en la destilacin fraccionada 3 ley de Henry 2 ley de Raoult 2 desviaciones de la ley de Raoult 3 elevacin del punto de ebullicin 2 depresin del punto de congelacin. Determinacin de masa molar 2 presin osmtica 2 coeficiente de particin 3 extraccin con disolventes 2 principios bsicos de cromatografa 2

336

337 338

339 340 341 342 343 344 345 346

TERMODINMICA
316 sistema y alrededores 2 317 energa, calor y trabajo 2 318 relacin entre entalpa y energa 2 319 capacidad calorfica (definicin) 2 320 diferencia entre CP y CV 3 321 ley de Hess 2 322 ciclo de BornHaber par compuestos inicos 3 323 clculo aproximado de energas de red (p. ej., ecuacin de Kapustinskii) 3 324 uso de las entalpas estndar de formacin 2 325 calores de solucin y solvatacin 2

347 348 349 350

Melbourne, Australia. Julio de 1998

xiii

30a Olimpiada Internacional de Qumica. Problemas preparatorios

OTROS
QUMICA ANALTICA
351 uso de la pipeta 1 352 uso de la bureta 1 353 eleccin de indicador para acidimetra 1 curvas de titulacin 354 pH (cidos fuertes y dbiles) 2 355 FEM (redox) 2 356 clculo del pH de un buffer sencillo 2 identificacin cualitativa 357 de Ag+, Ba2+, Cl-, SO421 358 de Al3+, NO2-, NO3-, Bi3+ 2 359 de VO3-, ClO3-, Ti4+ 3 360 K, Ca y Sr (identificacin a la flama) 1 361 Ley de LambertBeer 2

366 formas cis y trans 3

QUMICA TERICA
367 nmeros cunticos n, m y l 2 368 niveles de energa del tomo de hidrgeno (frmula) 2 forma de los orbitales p 2 configuracin espacial de los orbitales d 3 diagramas de orbitales moleculares molcula de H2 3 molcula de N2 o de O2 3 rdenes de enlace en O2, O2+, y O23 teora de Hckel para compuestos aromticos

369 370

371 372 373 374

COMPLEJOS
362 escritura de reacciones de complejacin 1 363 constantes de formacin (definicin) 2 364 trminos Eg y T2g: complejos octadricos de alto y bajo spin 3 365 clculo de la solubilidad de AgCl en NH3 (a partir Ks y las s) 3

3 375 cidos y bases de Lewis 2 376 cidos y bases duros y blandos 3 377 electrones desapareados y paramagnetismo 2 378 cuadrado de la funcin de onda y probabilidad 3 379 entender la ecuacin de Schrdinger ms sencilla 3

Melbourne, Australia. Julio de 1998

xiv

Problemas
PROBLEMA 1
Para muchos qumicos, los hidrocarburos con frmula emprica (CH) n resultan particularmente fascinantes. Estos compuestos inevitablemente presentan enlaces C C dobles o triples y/o ciclos para poder satisfacer las valencias de todos los tomos. Algunos ejemplos de hidrocarburos (CH)n son: n = 2: n = 6: n = 8: acetileno (etino) benceno cubano

n = 20: dodecaedrano

Consideremos ahora dos ejemplos con n = 4: el ciclobutadieno y el butatrieno. a) b) Dibuja las estructuras de estos dos ismeros de frmula C4H4. El enlace central C=C en el butatrieno tiene una longitud diferente a la de los otros dos. A qu se debe esto? (Sugerencia: considera la hibridacin de cada tomo.) El enlace central C=C en el butatrieno es ms corto o ms largo que los otros dos? Los cuatro tomos de carbono en el ciclobutadieno son equivalentes. Existe otra estructura con frmula C4H4 en la que todos los tomos de carbono son equivalentes y que tiene todas las valencias satisfechas. Este compuesto no ha podido ser aislado en el laboratorio a pesar de intensos esfuerzos por parte de diversos grupos de investigacin. Dibuja su estructura, y por analoga con algunos de los otros hidrocarburos (CH)n mencionados, propn un nombre trivial razonable para este compuesto. Cuntas formas monocloradas (C4H3Cl) existen del: i) ciclobutadieno? ii) butatrieno?

c)

d)

iii) e)

el compuesto descrito en la pregunta c)?

Cuntos ismeros diclorados (C4H2Cl2) existen del: i) ciclobutadieno? ii) butatrieno? iii) el compuesto descrito en la pregunta c)? Como podran ser identificados los ismeros del diclorobutatrieno, con base en los puntos de fusin?

f)

PROBLEMA 2
a) b) c) d) Dibuja un diagrama de niveles de energa que muestre como los orbitales 1 s de dos tomos de hidrgeno se combinan para formar los orbitales moleculares del H2. Describe los orbitales moleculares del H2 en relacin con los orbitales atmicos originales. Por qu al orbital de mayor energa en el H2 se le llama orbital antienlazante? De manera similar, podemos combinar los orbitales atmicos de tomos ms complicados para formar orbitales moleculares. Consideremos la molcula de oxgeno, O2. Acomoda los tomos de oxgeno como se muestra (a lo largo del eje y) y considera que cada tomo tiene orbitales 1s, 2s, 2px, 2py y 2pz. z O O y 1s 2s 2px 2pz

2py

Ahora construye los orbitales moleculares que surgen de la interaccin de los orbitales atmicos 2s, 2px, 2py y 2pz de los dos tomos de oxgeno y completa el diagrama mostrado a continuacin:

2pz

2py

h g z

2px

f e x y

2s

d c

1s

b a

O2

e)

Es posible mostrar estos orbitales moleculares en orden creciente de energa en un diagrama de orbitales moleculares:

h 2p Energa f e g 2s d c 1s b a j i

2p

2s

1s

O2

A qu se debe que la energa del orbital g sea menor que la de e o i, y la energa del orbital h mayor que la de f o j?

f) g)

Por qu tienen la misma energa los orbitales e e i? Si estirramos la molcula de O2 (es decir, si aumentramos la distancia OO), cmo cambiara la energa del orbital j? Sera este cambio mayor o menor que el cambio en la energa del orbital h?

PROBLEMA 3
Podemos aplicar la teora de orbitales moleculares para predecir algunas de las propiedades de CN, NN y NO. a) b) c) Cul es el orden de enlace para cada una de estas molculas? Del CN, N2 y el NO, cul tiene la energa de ionizacin (EI) ms grande? Cul tiene la energa de ionizacin ms pequea? Cul tiene la mayor afinidad electrnica? (La afinidad electrnica es la energa liberada al asociarse un electrn a una especie, y es positiva cuando este proceso es exotrmico.) El agregar o sustraer electrones del CN o del NO produce especies que son formalmente isoelectrnicas con el N2. Esperaras que estas especies isoelectrnicas tuvieran una fuerza de enlace tan alta como la del N2? Por qu?

d)

PROBLEMA 4
Durante mucho tiempo se crey que los gases nobles eran completamente inertes e incapaces de formar enlaces qumicos. Ahora sabemos que esto no es cierto, y ya la mayora de los libros de texto de qumica general describen algunos de los compuestos de xenn y kriptn que han sido obtenidos. a) b) c) Usando el principio de repulsin de los pares electrnicos de la capa de valencia, predice las geometras probables del XeF2 y XeF4. Cul es el nmero de oxidacin del xenn en estos compuestos? Esperaras que se comportaran como agentes oxidantes, o como agentes reductores? El helio es conocido ampliamente como el ms inerte de todos los elementos; sin embargo, aun la inercia del helio solamente es aplicable a su interaccin con otros tomos y molculas neutras. Los compuestos de helio, que involucran enlaces qumicos formales con otros tomos, slo pueden existir cuando la entidad resultante tiene una carga (generalmente positiva). Por ejemplo, el tomo de helio puede formar compuestos observables (aunque no necesariamente de larga vida) con H+, con He+ y con He2+. Utiliza la teora de orbitales moleculares para determinar el orden de enlace en cada uno de estos casos. Los dicationes diatmicos de frmula XHe2+ generalmente slo son posibles cuando EI(X+) < EI(He), es decir que la energa requerida para volver a ionizar a X+ es menor que la necesaria para ionizar al He. Sin utilizar una tabla de energas de ionizacin, identifica qu elemento (llammosle Z), entre el H y el Ar, es ms probable que cumpla con este requisito.

d)

e)

Cul de los vecinos (los que se encuentran inmediatamente arriba, abajo, a la izquierda o a la derecha) del elemento Z es ms probable que tambin forme un dicatin estable con el helio? Cul de los vecinos de Z es menos probable que forme tal compuesto?

PROBLEMA 5
El benceno es el prototipo de hidrocarburo aromtico. Algunos hidrocarburos ms grandes, que consisten en una red de anillos bencnicos (algo as como una pequea seccin de una lmina de grafito) tambin pueden tener carcter aromtico, y se conocen como hidrocarburos aromticos policclicos o PAHs. Muchas de las caractersticas estructurales de los PAHs pueden entenderse si se les considera compuestos de anillos aromticos bencnicos de 6 electrones (aunque tambin deberan ser considerados los sistemas de deslocalizacin ms grandes, como el 10 y el 14). En este problema supondremos que los orbitales 6 deslocalizados son suficientes para explicar las caractersticas estructurales de los PAHs. El PAH ms simple es el naftaleno, C10H8.

Ntese, que aunque el naftaleno suele dibujarse con ambos anillos totalmente deslocalizados, podemos esperar que sea menos aromtico que dos anillos de benceno separados. Esto se debe a que la deslocalizacin en un anillo pone restricciones en el otro anillo:

Aqu la deslocalizacin en el anillo marcado con lneas gruesas slo es posible si el otro anillo muestra una alternacin formal de enlaces sencillos y dobles C-C. Obviamente en una molcula simtrica como la de naftaleno no podemos esperar que los dos anillos tengan diferentes patrones de enlace, sino que deberamos decir que cada anillo tiene cierto carcter aromtico (mas no tanto como el de un anillo de benceno), y cierta tendencia a alternar enlaces sencillos y dobles. Si dibujamos el naftaleno como sigue: a b c a b

podemos ver que los enlaces a y c tienen un notable carcter de enlace sencillo, mientras que los enlaces b tienen un carcter de doble en lace (esto tambin es cierto para los enlaces anlogos en el otro anillo). Ahora consideremos los esqueletos hidrocarbonados de los siguientes tres PAHs:

H H H H H H H H H H H H H H H H H H H H H H H H H H H H H H H H

Fenantreno

Trifenileno

Coranuleno

a)

Usando

para denotar a los anillos que tienen carcter aromtico, y mostrando los

otros enlaces CC como sencillos o dobles, dibuja las formas de resonancia ms aromticas posibles para estos tres compuestos. La forma ms aromtica posible es aquella con el mayor nmero de anillos aromticos; para el naftaleno es b) .

Identifica, usando las letras L y C respectivamente, los enlaces carbonocarbono ms largos y ms cortos en el fenantreno, trifenileno y pireno. (Si un PAH no presenta enlaces inusualmente cortos o largos es decir, significativamente diferentes en longitud del enlace CC del benceno no los seales). Compara el nmero de anillos totalmente aromticos de las formas de resonancia obtenidas en a) con el nmero total de anillos en cada PAH. Con base en esto, cul es el ms aromtico de estos PAHs? Cul es el menos aromtico? Ahora compara el nmero de anillos totalmente aromticos con el nmero de tomos de carbono en cada PAH. Con base en esto, cul es el ms aromtico de estos PAHs? Cul es el menos aromtico? Ahora consideremos una lmina de grafito:

c)

d)

e)

i) ii) iii)

Cul es el orden de enlace CC promedio? Los enlaces CC son ms largos o ms cortos en el grafito que en el benceno? Es ms o menos aromtico el grafito que el benceno (por tomo de carbono)?

PROBLEMA 6
Una estudiante de doctorado recibi un pedido con los seis ismeros del C4H8 (que son gases a temperatura ambiente). Desafortunadamente, durante su transportacin se desprendieron las etiquetas de los tanques de gas y ya no puede identificarlos correctamente. Etiquet los tanques con las letras de la A a la F, y, en el intento de deducir el contenido de cada cilindro, hizo las siguientes observaciones: i) A, B, C, y D decoloran al bromo rpidamente (aun en la oscuridad), mientras que E y F no.

ii) iii) iv) v)

Los productos de la reaccin de B y C con Br2 son estereoismeros el uno del otro. A, B y C dan todos el mismo producto al reaccionar con H2 sobre un catalizador de Pd. E tiene un punto de ebullicin mayor que F. C tiene un punto de ebullicin mayor que B. Identifica los contenidos de los seis tanques.

PROBLEMA 7
Una de las reglas generales que ms ha durado en la Qumica es que prcticamente todas las reglas generales son violadas por algn compuesto u otro! Por ejemplo, el O2 viola la regla del apareo de electrones, y el PF 5 viola la regla del octeto. Otra regla para la que existen excepciones es el tan frecuentemente mencionado requisito para la isomera ptica en los compuestos orgnicos: estos compuestos deben tener un tomo asimtrico con hibridacin sp3 unido a cuatro grupos o tomos diferentes. Ahora se sabe que algunos de los fullerenos ms grandes, por ejemplo algunos de los ismeros del C76, son pticamente activos. Estas son molculas que formalmente slo tienen tomos con hibridacin sp2, por lo que cada tomo de carbono esta unido directamente a tan slo otros tres. Aqu la quiralidad surge del la curvatura de la red de tomos del fullereno y del patrn asimtrico de los anillos de 5 y 6 carbonos que la componen. a) b) c) Cuntos ismeros distintos existen del dicloroetileno? Seala los pares de estereoismeros. Qu clase de estereoisomera se observa para este compuesto? Cuntos ismeros diferentes existen del dicloropropadieno (tambin llamado dicloroaleno)? Qu clase de estereoisomera se observa en este caso? Enuncia una regla que prediga el tipo de isomera en funcin de n para los compuestos con la frmula general ClHC=(C=)nCHCl. Cul es el fundamento geomtrico detrs de esta regla?

PROBLEMA 8
Se analiz una muestra de dicloropropadieno en un espectrmetro de masas. Se observ una seal fuerte en el espectro de masas para una relacin masacarga (m/z) de 75, y otra en m/z = 77. Bajo ciertas condiciones de operacin, estas fueron las nicas seales visibles en el espectro. Bajo condiciones diferentes, la muestra produjo varias seales diferentes, incluyendo m/z = 82 (mas no 84) y m/z = 28 (pero no 27). Se encontr que, independientemente de las condiciones de operacin, la seal en m/z = 77 tiene el 60% de la intensidad de la seal en m/z = 75. Puedes suponer lo siguiente: Los iones observados son todos iones positivos con carga unitaria y surgen directamente de la ionizacin disociativa del dicloropropadieno, sin que ocurra ningn rearreglo durante la fragmentacin.

El dicloropropadieno fue preparado a partir de carbn elemental, hidrgeno y cloro por un mtodo no especificado: se sabe que los elementos utilizados tenan una composicin isotpica distinta de la que ocurre comnmente, pero slo contenan istopos estables. Adems, no se intent marcar ningn tomo especfico dentro de la molcula con algn istopo en particular.

a) b)

Cul es la frmula qumica de los iones detectados en m/z = 75 y 77? Cul es la distribucin isotpica observada en la muestra de dicloropropadieno? Calcula el porcentaje de cada isotopmero del dicloropropadieno. (Los isotopmeros son molculas que tienen frmula qumica idntica pero difieren en los istopos que las constituyen.) Cul es la masa molar de la muestra? Para simplificar asume que la masa atmica de cada nclido es idntica a su nmero de masa. Puedes identificar el ismero del dicloropropadieno estudiado aqu? Un reciente desarrollo de la espectrometra de masas tradicional es la ESMS (Electrospray Mass Spectrometry). La ESMS difiere de la espectrometra de masas tradicional nicamente en que se inyecta una solucin en el aparato y no hay fuente ionizadora. De este modo la tcnica detecta nicamente los iones que ya existen en la solucin. Por ejemplo, si se inyecta una muestra de bromuro de terbutilamonio (en un disolvente inerte) en un aparato de ESMS, se obtiene principalmente un pico en m/z = 242 en el modo de deteccin de iones positivos. En el modo de deteccin de iones negativos se obtienen dos seales de aproximadamente igual intensidad en m/z = 79 y m/z = 81.

c) d)

e)

Qu iones producen estas tres seales? Una muestra de isopropanol (2-propanol) inyectada en un aparato de ESMS muestra un pico en m/z = 61 como la seal ms intensa en el modo de deteccin de iones positivos. La seal ms intensa en el modo de deteccin de iones negativos ocurre en m/z = 59.

f)

Cules son los iones que dan lugar a estas dos seales?

PROBLEMA 9
El dilitio, sustancia imprescindible para el sistema de propulsin de la nave espacial de la Federacin Enterprise, es en realidad una especie conocida (aunque no presenta todas las propiedades que le han atribuido Roddenberry y colaboradores !). El dilitio se forma por la unin de dos tomos de litio en fase gaseosa: Li(g) + Li(g) Li2(g) a)

(1)

La entalpa de formacin del dilitio no es fcil de medir directamente. Sin embargo, se conocen los siguientes parmetros termoqumicos:

Para mayor informacin acerca del Enterprise, consulta las siguientes direcciones del WWW: http://startrek.msn.com/, http://startrek.simplenet.com/startrek/; http://www.cs.cityu.edu.hk/ckmau/engineer.html; o http://public.logica.com/stepneys/sf/filk/dilithium.html

Hf(Li(g)) = 159.4 kJ mol-1 EI(Li(g)) = 5.392 eV [1 eV = 96.486 kJ mol-1] D0(Li2+(g)) = 129.8 kJ mol-1 [D0 es la fuerza de enlace del Li2+(g)] EI(Li2(g)) = 5.113 eV Usando estos valores, determina el Hf(Li2(g)) y D0(Li2(g)). b) El qumico maestro en plasmas del Enterprise est probando el funcionamiento de las turbinas warp. Coloc 122.045 g de litio puro en la cmara de reaccin al vaco de la turbina de babor. La cmara de reaccin tiene un volumen de 5.9474 105 m3, y se mantiene en una temperatura de operacin de 610.25 K. Un monitor de presin de alta sensibilidad indic que la presin se estabiliz en 9.462 10-4 Torr (1 Torr = 0.133322 kPa); el anlisis espectrofotojergomtrico de la superficie interna de la cmara indic que todo el litio se haba evaporado. (Por supuesto, la cmara de reaccin est hecha de duranio, que tiene una presin de vapor de cero a 610.25 K.) Cules son las concentraciones de litio y dilitio en la cmara de reaccin? Cul es el valor de la constante de equilibrio, Kc, para la reaccin (1) a esta temperatura? c) Posteriormente, se colocaron 265.384 g de litio en una cmara de reaccin idntica (tambin a 610.25 K) y al vaco en la turbina de estribor. El medidor de presin de esta cmara se estabiliz en 1.0455 10-3 Torr. Cul es la presin de vapor del dilitio a 610.25 K? d) Suponiendo que la reaccin 2Li(g) Li2(g) es la nica fuente de energa para propulsin del Enterprise, calcula la masa mnima de litio que debe ser llevada como combustible para acelerar al Enterprise desde el reposo hasta la mitad de la velocidad de la luz. La masa total del Enterprise, que es una nave grande con varios cientos de tripulantes, es de 3.586 106 kg (excluyendo el combustible). Para simplificar los clculos, puedes asumir lo siguiente: los efectos relativistas pueden ser ignorados las cmaras de reaccin pueden mantenerse indefinidamente a 610.25 K sin ningn costo energtico el litio slido, colocado en la cmara de reaccin, puede vaporizarse sin ningn costo energtico (el proceso inverso tampoco libera energa) la entalpa del enlace LiLi puede convertirse con 100% de eficiencia en la energa cintica de la nave el litio llevado como combustible no necesita ser tomado en cuenta para la masa efectiva del Enterprise. (Evidentemente, algunas de estas suposiciones no son fsicamente razonables, pero eso no debe preocuparnos aqu. Estamos hablando de ciencia ficcin!) e) Finalmente, comenta acerca de qu tan apropiada es (o no) la formacin de dilitio como mtodo para viajar a velocidades cercanas a la de la luz. Sera una mejor alternativa la formacin de diberilio, Be2?

PROBLEMA 10
El anlisis elemental de una sustancia orgnica X mostr que tiene C (40.02% en masa) y H (6.75% en masa). No contiene cantidades detectables de N o S, y se asume que el resto de su composicin es oxgeno, el cual no es posible analizar confiablemente por mtodos directos. a) b) Cul es la frmula emprica de X? X es un lquido a temperatura ambiente. A 10 mL de X (densidad = 1.044 g mL-1) se le agreg ciclohexano, hasta tener un volumen total de 500 mL. Esta solucin tena una densidad de 0.777 g mL-1, y un punto de congelacin de +2.02 C. Tf(c-C6H6) = 6.60 C; Kf(c-C6H6) = 1.86 C kg mol-1. Cul es el peso molecular y la frmula de X, de acuerdo con estos resultados? X tambin es miscible con el agua. A 50 mL de X se le agreg agua ultrapura, tambin hasta un volumen total de 500 mL. Esta solucin tena densidad de 1.005 g mL-1 y un punto de congelacin de -3.54 C. K f(H2O) = 1.86 C kg mol-1. Cul es el peso molecular aparente de X a partir de estos datos? Se encontr que, en solucin acuosa, X reacciona con las bases. Se titularon 25.00 mL de la solucin acuosa preparada en c) con NaOH 1.247 M. El punto final, determinado con un pHmetro, se obtuvo con 33.60 mL de solucin de hidrxido. La solucin resultante, en el punto final, tena un volumen de 58.5 mL y una densidad de 1.003 g mL-1. El punto de congelacin fue de -2.78 C. Cul es la estructura del producto (Na+)iYi- que resulta de la reaccin de X con NaOH acuoso? Cul es la estructura del compuesto X? Explica cualquier discrepancia aparente entre las respuestas de las preguntas b), c) y d).

c)

d)

e)

PROBLEMA 11
Generalmente se considera que dos istopos tendrn reactividad qumica idntica; sin embargo, esto no es del todo cierto. La diferencia en reactividad de los istopos se debe a la dependencia que existe entre la energa vibracional de las molculas y las masas de los ncleos que las constituyen. Aunque los detalles de este fenmeno no nos preocupan en este problema, es importante notar que los compuestos que contienen istopos ligeros (p. ej. el 1H19F) tienen enlaces ligeramente ms dbiles que los de compuestos anlogos con istopos ms pesados (como el 2H19F). Este efecto isotpico habitualmente tiene poca o ninguna importancia a temperatura ambiente, pero puede ser muy importante en la qumica de bajas temperaturas. Un ambiente bastante estudiado que se caracteriza por tener temperaturas muy bajas (entre 10 y 20 K) es el que existe en las densas nubes interestelares , que son las grandes nubes de polvo y gas a partir de las cuales se forman finalmente las estrellas. La separacin de deuterio en las nubes interestelares ocurre por varios procesos, incluyendo el siguiente mecanismo: H2 + D HD + D HD + H D2 + H (1) (2)

Los parmetros termoqumicos relevantes de la reaccin (1) son: Hf(H2(g)) = 0 kJ mol-1 Hf(HD(g)) = 0.33 kJ mol-1 Hf(H(g)) = 216.00 kJ mol-1 Hf(D(g)) = 219.76 kJ mol-1 S(H2(g)) = 130.57 J K-1 mol-1 S(DH(g)) = 143.69 J K-1 mol-1 S(H(g)) = 114.60 J K-1 mol-1 S(D(g)) = 123.24 J K-1 mol-1

(Las entalpas de formacin son a cero kelvin [ms apropiadas a las temperaturas interestelares de lo que seran las mediciones a 298 K]; las entropas son a 298 K, pero para nuestros propsitos asumiremos que son independientes de la temperatura.) a) Determina el cambio en la energa libre G para la reaccin (1) a T = 20 K y T = 1000 K. En qu direccin es espontnea esta reaccin, si la cantidad inicial de todos los reactivos y productos es igual? b) Qu nos dice el signo de H (positivo o negativo) acerca de las fuerzas de enlace del H2 y HD? Qu nos dice el signo de S acerca de la reaccin (1) como est escrita y cul es la base fsica para el signo de S? Ahora consideremos la reaccin (2). Cul es el signo de H y S para la reaccin como est escrita? Asumiendo que los cambios de entalpa y entropa son iguales en magnitud a los encontrados en a), calcula el cambio de energa libre y determina la direccin espontnea a 20 K y a 1000 K. d) El hidrgeno molecular (en sus diversas formas isotpicas) se encuentra presente en concentraciones mucho mayores que el hidrgeno atmico (y deuterio atmico) en las nubes interestelares. Predice cul es la forma predominante del deuterio (D, HD o D 2) y del hidrgeno en las nubes interestelares, a una temperatura de 20 K, en cada uno de las siguientes casos: i) Cuando la abundancia csmica n(D) n(H) (es decir, cuando el nmero total de ncleos de deuterio, en cualquier forma qumica, es mucho menor que el nmero total de ncleos de hidrgeno), y ii) Cuando n(D) = n(H). El caso i) es el que ocurre realmente en el Universo.

c)

PROBLEMA 12
El helio es el nico elemento de la tabla peridica que fue detectado en un objeto extraterrestre (la corona solar) antes de haber sido aislado en el laboratorio. Ahora sabemos mucho acerca de las propiedades fsicas y qumicas del helio; pero durante casi treinta aos a partir de 1868, el espectro solar fue la nica fuente de informacin sobre este nuevo elemento. a) Con el conocimiento actual de la teora cuntica, este espectro tiene mucha informacin analizable. Por ejemplo, el espectro visible presenta una serie de lneas de absorcin en las siguientes longitudes de onda: 4338, 4540, 4858, 5410, y 6558 (1 = 10-10 m). El espaciado entre estas lneas indica que la absorcin es debida a la

excitacin de un tomo o ion hidrogenoide (es decir, que tiene la misma configuracin electrnica del H). Esta especie es He, He+, o He2+? b) Puede mostrarse que el nivel de energa comn a todas las transiciones involucradas en estas lneas de absorcin es el nivel de energa inferior n i = 4. A que niveles de energa superiores nf lleva cada lnea del espectro? Cul es el valor de la constante tipo Rydberg (es decir, la constante anloga a la RH del espectro atmico del hidrgeno) para la especie absorbente que sufre estas transiciones (Hei+)? La energa de ionizacin (EI) de una especie se mide frecuentemente en electrn volts (eV). Cul es la EI(Hei+)? Gracias a la espectroscopa atmica, se sabe que EI(He+) / EI(He) = 2.180. La suma de estas dos energas de ionizacin da la energa de aparicin, EA(He2+). La EA(He2+) es la energa que debe recibir un tomo de He para perder sus dos electrones. Calcula la frecuencia y la longitud de onda del fotn menos energtico que es capaz de lograr la doble ionizacin del helio. Es la luz del Sol, en la superficie de la Tierra, una fuente eficiente de estos fotones? Algunas constantes tiles: c = 2.997925 108 m s-1 h = 6.62618 10-34 J s 1 eV = 96.486 kJ mol-1 = 2.4180 1014 Hz

c) d)

PROBLEMA 13
Una estudiante (la misma desafortunada que mencionamos en el PROBLEMA 6) recibi un pedido de halogenuros alcalinos, pero adivina se desprendieron accidentalmente las etiquetas de todos los recipientes excepto uno: el del bromuro de potasio. El laboratorio donde ella trabaja no cuenta con instrumentos espectroscpicos, por lo que, en un intento para identificar las sales, utiliz una columna de intercambio inico. La resina de intercambio inico que seleccion es una resina de poliestireno del tipo cido fuerte. El polmero contiene grupos cido sulfnico, -SO3H, de los cuales nicamente el protn es intercambiable. Analiz las seis muestras desconocidas de halogenuros alcalinos (y el bromuro de potasio, para probar el mtodo) de la siguiente manera: Pes 5.00 0.01 g de cada muestra, y los disolvi en agua destilada en un matraz volumtrico de 100 mL. Pas 40 mL de cada solucin por la columna, recibiendo el eluyente en un matraz volumtrico de 250 mL. Enjuag la columna dos veces con agua destilada, la cual fue recibida en el mismo matraz, el cual llen finalmente hasta la lnea de aforo con ms agua destilada. Antes de pasar la siguiente muestra, recarg la columna usando suficiente HCl 1M, y enjuagando con agua destilada. Titul entonces alcuotas de 50 mL de cada solucin, por triplicado, usando hidrxido de sodio 3.26 10-2 M, usando fenolftalena como indicador. Sus resultados fueron los siguientes:

Muestra A B C D E F KBr

Volumen promedio 21.15 0.1 mL 29.30 0.1 mL 7.40 0.1 mL 21.20 0.1 mL 10.30 0.1 mL 29.15 0.1 mL 10.25 0.1 mL

En el anlisis de estos resultados, puedes suponer lo siguiente: Cada muestra tiene ms de 99% de pureza, Todos los recipientes que contenan las sales eran hermticos, y No hay dos recipientes con la misma sal; el pedido inclua fluoruros, cloruros, bromuros y yoduros, pero ningn compuesto de astato. a) b) c) Cul es la lgica que hay detrs del procedimiento usado por la estudiante? Escribe las ecuaciones de cualquier reaccin qumica involucrada. Cules de las muestras pueden ser identificadas con certeza a partir de este anlisis? Cules de las muestras slo pueden reducirse a dos o tres posibilidades? Usando el equipo disponible en el laboratorio vidrio de reloj, papel indicador, una solucin cida de persulfato de sodio (Na2S2O8), y un frasco gotero con solucin de almidn la estudiante logr identificar todas las sustancias desconocidas. Sin conocer los resultados de sus pruebas, explica cmo podran los materiales mencionados ser suficientes para identificar todas las muestras que no pudieron ser identificadas concluyentemente en b). Qu propiedad de los halogenuros alcalinos impidi la identificacin de algunas de las muestras por el mtodo de intercambio inico? Esperaras un problema similar si se tuvieran que identificar los halogenuros alcalinotrreos MX2?

d)

PROBLEMA 14
En ocasiones, la qumica puede causar ilusiones pticas cuando se estudian estructuras tridimensionales. Por ejemplo:

a) b)

Cuntos anillos de cuatro tomos de carbono tiene el cubano? Para un hidrocarburo CnHm, Cmo se relaciona m, el nmero de tomos de hidrgeno con n, el nmero de tomos de carbono, r, el nmero de anillos, y d, el nmero de dobles enlaces?

c) d)

De acuerdo con la respuesta dada en a), cul es la frmula del cubano? Es esta la frmula correcta? Como distraccin momentnea, consideremos los siguientes hidrocarburos aromticos policclicos (PAHs): el coroneno y el coranuleno. Estos compuestos tienen, respectivamente, los siguientes esqueletos de carbono.

Ambos compuestos tienen el mximo de enlaces carbonocarbono conjugados. Redibuja el coroneno y el coranuleno, asegurndote de que estn completamente conjugados (incluyendo la conjugacin de los tomos del polgono central). Ahora, a partir del nmero de tomos de carbono, anillos y dobles enlaces en la molcula, intenta predecir el nmero de tomos de hidrgeno en cada molcula, y dibuja los hidrgenos necesarios para satisfacer todas las valencias faltantes de los tomos de carbono. Es igual el nmero de tomos de hidrgeno agregados igual al predicho usando la frmula encontrada en la pregunta b)? e) Los dos PAHs mostrados arriba se muestran como si fueran planos. En realidad, uno es plano, mientras que el otro no (aunque no est demasiado desviado de la planaridad). Cul de ellos es plano y cul no? Ahora, usando el dibujo plano del PAH plano como inspiracin, dibuja el cubano como si fuera plano. No intentes introducir ninguna perspectiva, pero s conserva la integridad de todos los enlaces CC y HH del cubano. El dibujo producido puede verse bastante distorsionado (como debe ocurrir siempre que se dibuja una estructura sumamente tridimensional como si ocupara solamente dos dimensiones), y no debe mostrar de ninguna manera un enlace cruzando sobre otro. A partir de esta proyeccin plana, cuenta el nmero de anillos en la molcula. Est esto de acuerdo con la respuesta de a)? Y con la respuesta de b)? Una prueba ms: determina el nmero mximo de enlaces CC que pueden ser rotos en el coroneno y en el coranuleno sin partir la molcula en fragmentos independientes (para esta pregunta no diferencies entre los enlaces dobles y los sencillos). Redibuja el esqueleto de carbono de estos PAHs, sin estos enlaces, para asegurarte de que todos los tomos de carbono estn todava conectados. Hay alguna relacin entre el nmero de enlaces que pueden ser rotos y el nmero de anillos? De ser as, cul es esta relacin? Cuantos enlaces pueden ser rotos en el cubano sin fragmentar el esqueleto de carbono? Se obtiene un nmero de anillos r igual al de a), b), o f)?

f)

g)

h)

i)

Ahora consideremos el dodecaedrano, C20H20:

Dibuja una versin plana de esta molcula, anloga a la dibujada en f). Cuntos anillos tiene esta molcula de 12 caras? Est este nmero de acuerdo con la frmula encontrada en b)? j) Por ltimo, consideremos el buckminsterfullereno, C60:

Para esto no es necesario intentar hacer un dibujo plano (suspiros de alivio ), pero es s hay que hacer algunos clculos. Cada tomo de carbono en el C 60 tiene tres vecinos inmediatos. Cuntos enlaces carbonocarbono hay en la molcula? (Si tienes problemas, recuerda el caso del cubano, donde cada tomo de carbono tiene tambin tres vecinos inmediatos.) Cuntos de estos enlaces deben ser dobles para satisfacer todas las valencias? Usando la frmula encontrada en b), calcula cuntos anillos debe tener el C60. Euler demostr que cualquier red cerrada formada solamente de pentgonos y hexgonos debe tener exactamente doce pentgonos independientemente del nmero de hexgonos. Puedes afirmar concluyentemente cuntos de los anillos del C60 son pentgonos y cuntos son hexgonos?

PROBLEMA 15
La acidez de una muestra de agua es afectada por la absorcin de gas. El gas ms importante en este sentido es, generalmente, el dixido de carbono. a) b) Escribe las tres ecuaciones qumicas que muestran el efecto del CO 2 atmosfrico sobre la acidez del agua. Ordena las siguientes mezclas gaseosas segn su tendencia a disolver el CO 2(g) en agua (los porcentajes son % en mol): i) 90% Ar, 10% CO2 ii) 80% Ar, 10% CO2, 10% NH3 iii) 80% Ar, 10% CO2, 10% Cl2 Escribe las ecuaciones de cualquier reaccin qumica que ocurra en solucin acuosa debido a la exposicin a estas mezclas de gases.

c)

Ordena los siguientes sistemas acuosos segn su capacidad para disolver el CO 2. Puedes asumir que antes de ser expuestos a CO2 al 10% en Ar, los sistemas estuvieron en equilibrio con el aire. i) Agua destilada ii) HCl 1 M iii) Acetato de sodio 1 M Suponiendo un contenido de CO2 de 350 ppm (en volumen) en la atmsfera, y suponiendo que ya se ha alcanzado el equilibrio entre el CO 2 acuoso y el gaseoso, calcula el pH de una gota de lluvia a presin atmosfrica. Las constantes apropiadas a 25 C son: KH(CO2) = 3.39 10-2 mol L-1 atm-1, Kb(HCO3-) = 2.24 10-8, Kb(CO32-) = 2.14 10-4. Calcula el pH del agua carbonatada embotellada (P(CO2) = 1 atm). Una muestra de 100 mL de agua de lluvia es titulada con NaOH 1.00 10-4 M (en agua destilada y descarbonatada) bajo condiciones experimentales que impiden un posterior intercambio de gases. Traza la curva de valoracin. La grfica debe indicar el pH esperado al principio de la titulacin; a 7.1 mL; y a 21.2 mL de solucin de hidrxido de sodio agregada.

d)

e) f)

PROBLEMA 16
a) Cuntos cidos carboxlicos existen con la frmula condensada C5H10O2? Dibuja y nombra todos estos compuestos. (No nombres los ismeros pticos, ni dibujes los enantimeros por separado, pero s indica, con asteriscos en los carbonos quirales, cules estructuras son pticamente activas.) Ordena los cidos carboxlicos mencionados decrecientemente segn su valor esperado de Ka en solucin acuosa. Explica tus razones. Cuntos de los ismeros estructurales con frmula RCOOH, donde R = C4H8F, presentan actividad ptica? Dibuja estos ismeros estructurales, identificando los tomos quirales, e intenta predecir cul es la especie ms cida y cul es la menos cida. Justifica tu razonamiento. Qu producto orgnico se espera de la reaccin de un cido carboxlico con un alcohol bajo condiciones cidas? Escribe una ecuacin generalizada para esta reaccin, mostrando los reactivos y los productos. Si se hace reaccionar propanol marcado con 18O con cido etanoico no marcado y en medio cido, se encuentra que el istopo marcado se encuentra solamente en el producto orgnico. Dibuja este producto, y escribe un mecanismo generalizado para la reaccin del alcohol con el cido carboxlico. Si en la reaccin se utiliza propanol no marcado y cido etanoico marcado (
CH3C OH
18

b) c)

d)

e)

f)

), muestra los productos esperados:

i) ii)

para la reaccin en HCl 1 M, y para la reaccin en NaOH 1 M, seguida por una acidificacin con DCl 1 M.

PROBLEMA 17
Los gases nobles, y en particular el helio, no suelen ser considerados como objeto de estudio para la qumica del estado slido. Sin embargo, se conocen algunos materiales que contienen tomos de helio atrapados o incrustados. Las molculas que son capaces de aprisionar a los tomos de los gases nobles se conocen como clatratos. Estos compuestos se pueden encontrar en meteoritos, los cuales liberan una pequesima cantidad de gases nobles al ser calentados. Una comparacin entre la respuesta trmica de la (hasta ahora desconocida) fase del meteorito que contiene los gases nobles con la de la especie He@C 60 (la @ indica que el helio est contenido en la jaula de fullereno C60 pero no est enlazado qumicamente) ha llevado a sugerir que fullerenos presentes en los meteoritos pueden ser los responsables de que haya gases nobles atrapados. a) b) El dimetro de la jaula C60 del fullereno es de 7.0 (1 = 10 -10 m). Calcula el volumen interior del C60, y la presin interior en la especie He@C60, a 298 K. En 1994, se report el resultado del anlisis de contenido de fullereno en el material del lugar donde cay un meteorito en Sudbury, Ontario, hace 1850 millones de aos. Se sospech que el C60 encontrado pudo formarse qumicamente en un proceso debido al impacto. En 1996 se report que algunos de los fullerenos extrados del material contenan helio, obtenindose los siguientes resultados: 1.15 10-7 cm3 de 3He (TPS) por g de C60 2.09 10-4 cm3 de 4He (TPS) por g de C60 Qu fraccin de las molculas de C60 en la muestra contienen un tomo de 3He? Cul es la presin parcial media efectiva de 3He y 4He dentro de la muestra? c) El contenido de 4He en la atmsfera terrestre es actualmente de 5.240 0.004 ppm de volumen. Si se asume que la presin parcial de 4He en la muestra de C60 refleja la presin parcial de helio en la atmsfera cuando se form el fullereno, y que la cantidad de helio en la atmsfera prcticamente no ha cambiado en los ltimos 2000 millones de aos, concuerdan los resultados obtenidos con un origen terrestre del fullereno? d) La relacin 3He/4He en la atmsfera terrestre es de 1.3 10-6; el valor de esta relacin en materiales de la corteza terrestre suele ser aun menor. Concuerda el valor de 3 He/4He observado en la muestra de fullereno con un origen terrestre? e) Ahora, para variar: Consideremos un fullereno puramente hipottico con frmula C5.010 , que consiste en una capa sencilla y esfrica de carbono, con una presin
9

interior de helio de 120 kPa. Asumiendo que la distancia CC es de 1.42 , y que la superficie es como una lmina de grafito, calcula: i) el rea superficial, el dimetro, y el volumen interior de este fullereno gigante, ii) la densidad del fullereno lleno de helio, iii) la densidad del aire a temperatura y presin estndar (asumiendo que M r(aire) = 29.0 g mol-1). iv) Comenta sobre las implicaciones de ii) y iii) sobre las propiedades fsicas del fullereno.

PROBLEMA 18
Los radioistopos pueden utilizarse en medicina nuclear de dos maneras. La radioterapia consiste en atacar los sitios de divisin celular activa con radionclidos para inducir la muerte de las clulas. La imagenologa nuclear usa los radioistopos para revelar los detalles metablicos de un rgano. Una de estas tcnicas es la que se utiliza para determinar el volumen sanguneo de un paciente. a) Tres compuestos radiofarmacuticos contienen, respectivamente, los radioistopos 71 Zn (t1/2 = 2.4 minutos), 67Ga (t1/2 = 78.25 horas) y 68Ge (t1/2 = 287 das), cada uno con una actividad de 7.0 107 Bq/mL. Para cada compuesto: i) ii) b) c) Calcula la actividad por mL despus de 30 minutos, y Calcula la actividad por mL despus de 30 minutos y una dilucin de 10 mL del producto radiofarmacutico a un volumen de 25 L.

Ignorando los efectos qumicos, cul es la ventaja que tiene el 67Ga sobre los otros dos radioistopos para la determinacin del volumen sanguneo? Los modos de decaimiento de estos istopos son: emisin de partcula - (71Zn) y captura electrnica (67Ga y decaimiento?
68

Ge). Cules son los productos de estos procesos de

d)

Un QFB prepar citrato de galio (GaC6H5O63H2O) a partir de una muestra de galio enriquecido en 67Ga (5.0 10-5 % en mol de 67Ga; 10.25 mg de Ga total). La sntesis del citrato de galio es cuantitativa; una vez sintetizado, el producto se disolvi en 100 mL de agua. Ocho horas despus de que se fabric el 67Ga, se le administr intravenosamente 1 mL de la solucin a un paciente, y una hora despus se le tom una muestra de sangre de 1 mL. i) Calcula la actividad en Bq de la dosis de 1 mL de solucin de citrato de galio. ii) Si la muestra de sangre tiene una actividad de 105.6 Bq, cul es el volumen sanguneo del paciente?

PROBLEMA 19
Las siguientes semirreacciones se encuentran relacionadas con la especiacin del uranio en solucin acuosa. U3+ + 3eU4+ + eUO22+ + eUO22+ + 4H+ + 2eUO22+ + 4H+ + 6eUO2+ + 4H+ + ea) b) U U3+ UO2+ U4+ + 2H2O U + 2H2O U4+ + 2H2O E = -1.798 V E = -0.607 V E = -0.062 V E = +0.327 V E = -1.444 V E = +0.620 V

Asigna estados de oxidacin a las diversas especies que contienen uranio en las medias reacciones anteriores. Analizando las medias reacciones anteriores, determina el destino qumico final de un pequeo fragmento de uranio puesto en contacto con una solucin 1 molar de un

cido fuerte monoprtico y una presin de hidrgeno de 1 atm, todo a 25 C. Proporciona las reacciones balanceadas y los potenciales de electrodo para todas las reacciones. (Puede asumirse que la base conjugada X- no reacciona de manera perceptible con el uranio o sus compuestos.) c) d) Cul es la especie de uranio ms estable a pH = 6 (y las dems condiciones estndar)? Determina el rango de pH, para soluciones cidas o neutras, en el que una solucin 1 M de UO2+ sera estable: i) con las dems condiciones estndar (P(H2) = 1 atm, concentraciones de las dems especies de uranio = 1 M). ii) con P(H2) = 1.0 10-6 atm, y las dems condiciones estndar. Cules de estas condiciones son ms adecuadas para la especiacin del uranio en aguas naturales?

PROBLEMA 20
Un recipiente de vidrio sellado, que tiene dos filamentos de tungsteno separados por un espacio de 5 mm, es llenado con aire limpio y seco a temperatura y presin estndar. Se establece una descarga elctrica entre los filamentos. Durante los siguientes minutos, se ve que el gas dentro del recipiente toma un color caf caracterstico. a) b) Cul es la especie responsable de la coloracin observada? Estima un lmite mximo para su concentracin en el recipiente. El mismo color caf se desarrolla espontneamente cuando se introducen oxgeno y xido ntrico en un bulbo de vidrio vaco. Escribe la ecuacin global de la reaccin que ocurre en el bulbo. Las siguientes mediciones fueron obtenidas de varios experimentos, realizados a una temperatura de 25 C: [NO] (mol L-1) 1.16 10-4 1.15 10-4 1.18 10-4 2.31 10-4 5.75 10-5 i) ii) d) [O2] (mol L-1) 1.21 10-4 2.41 10-4 6.26 10-5 2.42 10-4 2.44 10-5 Rapidez inicial (mol L-1 s-1) 1.15 10-8 2.28 10-8 6.24 10-9 9.19 10-8 5.78 10-9 A ( = 400 nm) 0.341 0.331 0.335 0.656 0.166

c)

Determina el orden de reaccin con respecto a O2, a NO, y el orden global. Determina la constante de rapidez de reaccin a 298 K.

La columna en el extremo derecho de la tabla anterior muestra la absorbancia a 400 nm (longitud de la celda = 10 cm) cuando ha pasado suficiente tiempo para que la mezcla de reaccin alcance el equilibrio. i) Se lleva a cabo de manera prcticamente completa la reaccin? ii) Determina el coeficiente molar de absorcin para la longitud de onda medida.

iii) e)

La longitud de onda de mxima absorbancia estar cerca de 400, 500 600 nm?

Cuando el gas obtenido en esta reaccin se aisl en un aparato cuyo volumen puede ser modificado con un pistn, y se obtuvieron los siguientes resultados: V (mL) 1000 500 200 100 50 20 10 Ptot (atm) 2.49 10-3 4.90 10-3 1.18 10-2 2.25 10-2 4.23 10-2 9.60 10-2 1.78 10-1

La temperatura del aparato se mantuvo a 25 C durante todas las mediciones. Escribe una ecuacin de reaccin que explique la dependencia de la presin con respecto al volumen, y determina la constante de equilibrio correspondiente. f) Al someter el sistema descrito en e) a una compresin mayor, se obtuvieron los siguientes resultados: V (mL) 10 5 2 1 0.5 0.2 Ptot (atm) 0.178 0.331 0.765 1.215 1.215 1.215

Por qu deja de aumentar la presin con volmenes de 1 ml o menos? Escribe las ecuaciones de cualquier reaccin qumica que ocurra, y calcula la presin parcial de cualquier especie gaseosa para V = 1 mL. Cul es la constante de equilibrio para el proceso responsable del tope de presin de 1.215 atm?.

PROBLEMA 21
La investigacin de las reacciones de complejacin de los iones de los metales de transicin Mn+ + mL MLm(n-m)+

frecuentemente se complica debido a la existencia de equilibrios colaterales. Por ejemplo, es muy comn que el ligante L- sea la base conjugada de un cido dbil, por lo que su concentracin en solucin depende marcadamente del pH. En estos casos es comn modificar la expresin de la constante de formacin del complejo, m: m = [MLm(n -m)+ ] [M n+] [L ]m

sustituyendo en lugar de [L-], el parmetro L-CT(L), donde CT(L) es la concentracin total de L en solucin todas sus formas (ya sea como HL, como L - o como MLi(n-i)+) y L- es la fraccin de L total que se encuentra en la forma L -. Este mtodo se usa

frecuentemente en el anlisis por titulacin con EDTA, ya que el EDTA (H4Y) es un cido tetraprtico dbil que slo es capaz de formar complejos eficientemente cuando se encuentra en su forma totalmente desprotonada Y4-. Puede demostrarse que Y4tiene la forma Ka,1 Ka,2 K a,3 Ka,4 Y4- = [H +]4 + Ka,1 [H+]3 + Ka,1 Ka,2[H +]2 + Ka,1 Ka,2 Ka,3[H +] + Ka,1 Ka,2 Ka,3 Ka,4

donde Ka,i es la i-sima constante de disociacin del EDTA (los valores son, respectivamente, de 1.02 10-2, 2.14 10-3, 6.62 10-7, y 5.50 10-11). a) Determina el valor del parmetro Y4- a pH de 2, de 6 y de 10. Cul es la concentracin del anin completamente desprotonado Y4-, en 500 mL de solucin que contienen 3.252 g de EDTA, para cada valor de pH? b) La constante de formacin KY, para la complejacin de Mn+ con Y4-, tiene un valor de 6.3 1021 (Hg2+), 2.1 1014 (Fe2+), y 5.0 1010 (Ca2+). Cul de estos iones metlicos formar complejos con el EDTA con un rendimiento mayor a 99.9%, en una solucin con 5.00 10-3 mol L-1 de EDTA total, en un pH amortiguado en: i) ii) iii) c) 2? 6? 10?

El ion mercrico, Hg2+, tiene una gran afinidad con los cloruros: Hg2+ + 4Cl- HgCl42-

La constante de formacin Cl = 3.98 1015. Para una solucin que tiene 0.5 mol L -1 de cloruro total y 5.00 10-3 mol L-1 de EDTA total, determina la fraccin de Hg que existe como el ion libre, como el anin tetracloromercurato, y como el complejo con EDTA, a pH: i) 2. ii) 6. iii) 10. (Puedes asumir que la concentracin total del metal es mucho menor a 0.05 mol L-1.) d) Se tiene una amalgama que se sabe que contiene nicamente mercurio, sodio y calcio. Se trataron 5.218 gramos de la muestra con un agente oxidante apropiado, y se llevaron a un volumen de 500 mL. Se titularon alcuotas de 25 mL de esta solucin, en un pH amortiguado en 2.6, con una solucin de MgY 2- 0.0122 mol L-1. El volumen gastado promedio fue de 44.19 mL. Cuando se titularon alcuotas de 10 mL con un pH amortiguado en 9.5, se gastaron en promedio 57.43 mL. Determina el porcentaje en masa de mercurio, sodio y calcio en la amalgama.

PROBLEMA 22
Las emisiones de gases de invernadero son motivo de preocupacin por sus efectos en el ambiente a nivel global. El aumento en la concentracin de un gas de invernadero, el CO2, ha sido bastante estudiado en las ltimas dcadas.

a)

Aproximemos la distribucin de los gases de invernadero suponiendo que estos gases (CO2, vapor de agua, etc.) se encuentran ubicados en una capa en una altitud entre 10 y 11 km. (Esta descripcin no realmente correcta, pero sirve para ilustrar algunos conceptos.) Cul es el efecto de esta capa de gases de invernadero sobre la temperatura de la atmsfera a una altitud de 5 km? Cul es el efecto de esta capa sobre la temperatura atmosfrica a una altitud de 15 km? Explica cualitativamente este efecto colateral de la capa de gases de invernadero. Cmo afectar el cambio de temperatura en la atmsfera baja a los equilibrios del CO2 y del H2O entre las fases gaseosa y acuosa? Un cambio en la posicin de equilibrio de estas especies tendr algn efecto, por s mismo, sobre la temperatura de la atmsfera baja? Ahora pasemos de la capa de los gases de invernadero a la capa de ozono. El ozono se produce y se destruye de manera fotoqumica. Propn un mecanismo que explique la produccin y destruccin de ozono en una atmsfera de oxgeno puro. Las entalpas de formacin del O y O3 son Hf(O) = 249 kJ mol-1 y Hf(O3) = 143 kJ mol-1. Determina la longitud de onda mxima de los fotones capaces de producir la fotlisis del oxgeno y del ozono, respectivamente.

b)

c)

d)

e)

f)

En las regiones polares, se observa durante la primavera una gran disminucin en los niveles de ozono estratosfrico. Se piensa que las condiciones para esta disminucin catastrfica (llamada agujero de ozono) son favorecidas por varios factores, entre ellos los siguientes equilibrios: HCl(g) + NPE ClONO2(g) + NPE ClO(g) + ClO(g) HClNPE ClONO2NPE ClOOCl (1) (2) (3)

NPE denota una nube polar estratosfrica, que resulta de la condensacin de vapor de agua y otras sustancias voltiles en altitudes elevadas. La formacin de NPE es comn en la estratosfera antrtica durante el invierno y principios de la primavera, pero es menos comn el en rtico donde las temperaturas no son tan bajas. Se sabe que el agotamiento del ozono es ms intenso cuanto ms a la derecha se encuentren estos equilibrios. Tomando en cuenta que la formacin de enlaces es generalmente exotrmica, cmo sern afectados por la temperatura los equilibrios anteriores? g) La formacin de un agujero de ozono en la estratosfera rtica es un fenmeno que apenas ha sido notado recientemente: originalmente se crey que el rtico sera inmune a la formacin de un agujero de ozono. Evidentemente no fue as. Basndote en la informacin dada anteriormente, sugiere cual de las siguientes estrategias podra explicar la formacin del agujero de ozono en el rtico: i) Los niveles de CFCs en la estratosfera del hemisferio norte apenas recientemente han alcanzado los niveles que se tienen en la Antrtida desde hace de una dcada; o

ii) iii) iv)

El aumento de la concentracin de gases de invernadero en la atmsfera baja ha reducido la temperatura de la estratosfera rtica; o Un aumento en la concentracin de vapor de agua en la estratosfera rtica ha hecho ms favorable la formacin de NPE de lo que era anteriormente; o Un aumento en la cantidad de radiacin IR que llega a la estratosfera rtica est causando una mayor fotlisis del ozono rtico.

PROBLEMA 23
Una sustancia Q de abeja reina contiene 65.2% de carbono, 8.75% de hidrgeno y no tiene ningn otro elemento excepto oxgeno. Se sabe que Q es cido y la titulacin de 43.7 mg de este compuesto requiri 23.7 mL de hidrxido de sodio 0.0100 M para llegar al punto de equivalencia. Se determin adems que el peso molecular de Q es menor a 200. a) Cul es la formula molecular de Q y cules son los grupos funcionales que pueden ser responsables de la acidez del compuesto? Q reacciona con el hidrgeno en presencia de platino finamente dividido, dando un compuesto A. Una reduccin posterior de A con borohidruro de sodio en etanol da una sustancia B. El compuesto B se deshidrata fcilmente al calentar con cido sulfrico concentrado, dando un alqueno C. La resonancia magntica nuclear de 13C revel, entre otras cosas, que el compuesto C tiene un grupo metilo unido a un doble enlace. b) Qu grupos funcionales concuerdan con las reacciones descritas? La ozonlisis de C seguida por una oxidacin dio solamente dos fragmentos: cido actico y un cido dicarboxlico de cadena lineal D. Un tratamiento similar para Q dio cido oxlico (cido etanodioico) y una sustancia E que contiene un grupo cido carboxlico. c) Deduce las estructuras de D y E, y con ello da estructuras posibles para Q.

PROBLEMA 24
El cido crisofnico es un pigmento natural derivado de la antraquinona, que ha sido aislado de la raz de ruibarbo y tiene la estructura que se muestra a continuacin. Una sntesis muy elegante de esta molcula fue concebida en la Escuela de Investigacin en Qumica de la Universidad Nacional Australiana.
OH O OH

CH3 O cido crisofnico

Se redujo el 3-metilanisol (3-metil-metoxibenceno) con litio metlico en una mezcla de amoniaco lquido, tetrahidrofurano y ter-butanol, para dar el compuesto B

(C8H12O). Al tratar a B con amiduro de potasio en amoniaco lquido, y posteriormente con agua, se isomeriz para dar C. a) Dibuja las tres estructuras posibles de C. El espectro de RMN de 1H de C revel, entre otras cosas, la presencia de dos protones olefnicos que no son adyacentes entre s. Adems revel la presencia de dos grupos metileno vecinos, uno de los cuales es vecino a un protn olefnico. b) Dibuja las estructuras de C que sean consistentes con esta informacin. La reaccin de C con el 5-hidroxi-naftalen-1,4-diona dio el producto de adicin de Diels-Alder D (C18H18O4). El espectro de RMN de 1H de D mostr una seal en 10.5 que integra para un protn y que indica la presencia de un grupo hidroxilo enlazado intramolecularmente (por medio de un puente de hidrgeno). c) Sugiere tres estructuras posibles para el compuesto D. La enolizacin de D por medio de carbonato de potasio en metanol caliente, seguida de una oxidacin con nitrodisulfonato de potasio (sal de Fremy) dio un producto quinonoide de color amarillonaranja E (C18H16O4). El espectro de RMN de 13C de E tena en total 9 resonancias atribuibles a carbonos cuaternarios. La pirlisis de E a 180 C durante 15 minutos dio como producto F (C16H12O4). El espectro de RMN de 1 H F mostr, entre otras cosas, tres singuletes correspondientes a un protn cada uno (el ms bajo de ellos estaba en 11.0) y dos singuletes de tres protones cada uno, en 4.01 y 2.25. d) Con base en esta evidencia, sugiere estructuras posibles para los compuestos E y F. Cuando se trat a F con tricloruro de boro en diclorometano a -10 C, dio, despus del tratamiento posterior, un slido de color anaranjado. El espectro de masas de este producto mostr al ion molecular en m/z = 254. Se comprob que este producto es idntico al cido crisofnico natural. e) Dibuja el conjunto completo de estructuras, trazando el esquema de la sntesis total del cido crisofnico.

PROBLEMA 25
Los llamados rearreglos de Claisen son herramientas tiles para el qumico orgnico sinttico. Sin embargo, para el estudiante, estas reacciones son frecuentemente difciles de reconocer y requieren cierta reflexin para ser entendidas. Un ejemplo prctico de esta reaccin es el que se muestra a continuacin. Estdialo por un momento e intenta resolver el siguiente problema, que entre otras cosas, involucra un ejemplo de esta reaccin.
R O OMe calor R O OMe

En 1977 fue aislado un sesquiterpeno furano (K) a partir de un coral australiano Sinularia gonatodes. Se encontr que este compuesto es capaz de contrarrestar el veneno de abeja, y el hecho de que un compuesto tan simple pueda actuar como antiinflamatorio llam la atencin de los qumicos sintticos. A continuacin describiremos una de las sntesis que se han desarrollado. El cido A fue esterificado con el 2-(trimetilsilil)etanol (Me3SiCH2CH2OH), y el cetoster resultante fue reducido selectivamente en el carbonilo de la cetona con NaBH4/CeCl3, para dar el compuesto B (C14H22O4Si).
HO 2C

O O A

Al tratar a B con 1,1,1-trimetoxietano en presencia de trazas de un cido anhidro, se obtuvo un compuesto intermediario que sufri un rearreglo de Claisen para dar el producto C. El anlisis espectroscpico de C revel la presencia de un ster de trimetilsililetilo y un ster metlico. a) Deduce las estructuras de B y C, as como la del intermediario que lleva a C. La reduccin de C con borohidruro de litio dio un alcohol primario D (C16H26O4Si), el cual fue oxidado con clorocromato de piridinio para dar F. La reaccin de F con el reactivo de Wittig G dio dos ismeros H e I. El producto principal H tiene estereoqumica (E).
Ph3P G CHO

b)

Da las estructuras de los compuestos D-I, teniendo el cuidado de mostrar la estereoqumica correcta para los ismeros H e I. Una ltima reaccin de Wittig de H con el iluro derivado del yoduro de metiltrifenilfosfonio dio J, el cual dio K despus de romper el ster de trimetilsililetilo con fluoruro de tetra-n-butil amonio.

c)

Da todas las estructuras de esta secuencia sinttica.

PROBLEMA 26
Nota a los mentores: Las siguientes reacciones son las que quedan fuera de la qumica normal de nivel medio, y pueden servir para cualquiera de los problemas de la 30 a Olimpiada Internacional de Qumica. Aunque conocer el mecanismo puede ser til para el estudiante, los mecanismos no se consideran imprescindibles para resolver los problemas. Sin embargo, se asume que el estudiante conoce la regioqumica y la estereoqumica de estas reacciones.

REDUCCIONES (Asume un tratamiento posterior con agua a con pH ajustado.) (i) Li/NH 3 /tBuOH

OCH 3 (ii) Na/NH3 /EtOH CO2 H (iii) O Zn-Hg/H+

(iv)

CHO NH 2 NH 2 /OH-

(v)

CO2 Me H2 /Pd
HN=NH
(Nota: la diimida HN=NH se genera por oxidacin de la hidracina)

(vi)

COOH

(vii)

O LiAlH4 /AlCl3 O NaBH4

(viii)

OXIDACIONES (ix) KMnO4 /H2 O/OH

(x)

H+ / Cr2O 72- / acetona OH

(xi)

OH

clorocromato de piridinio CH2Cl2

(xii)

(i) O3 (ii) Me2 S

(xiii)

(i) O 3 (ii) KMnO 4

(xiv)

cido m-peroxibenzoico CH2Cl2

(xv)

(i) B2 H6 (ii) H2 O2 /OH-

(xvi) OSO 4 /Me3 NO OTRAS REACCIONES CO 2 H (xvii) CO 2 H (xviii) HO 2 C (xix) CHO Ph3 P O Ph3 P CH CO2 Me CH2 CH3 CO2 H

(xx)

(xxi)

O CHO (EtO) 2 PCH 2 CO 2 Et/NaH

(xxii) O

CO2 Me CO2 Me

(xxiii)

O O O

(xxiv)

OH

O O CH3COCCH3 piridina

(xxv)

OH

cido actico / H+ reflujo PhS -

(xxvi)

OCH 3 (xxvii) OCH3 O

BBr3 /CH2 Cl2 OCH3 (xxviii) CH3 CO2H CO2H anhdrido actico

Notas adicionales: Con la excepcin de (xiii) y (xvi), se asume que el estudiante podr dar el nombre segn la IUPAC de la materia prima. Con excepcin de (xiii), (xiv), (xvii), (xx), (xxii) y (xxiii) se espera lo mismo para los productos.

PROBLEMA 27 - EXPERIMENTAL
Sntesis del [Co(NH3)5Cl]Cl2 y de los ismeros de enlace [Co(NH3)5ONO]Cl2 y [Co(NH3)5NO2]Cl2. a) Preparacin del [Co(NH3)5Cl]Cl2. Disuelve 10.0 g de cloruro de amonio en 60 mL de amoniaco concentrado (PRECAUCIN!) en un matraz Erlenmeyer de 500 mL. Agitando la solucin

continuamente con un agitador magntico, agrega 20 g de cloruro de cobalto hexahidratado finamente pulverizado. (CUIDADO: evita la exposicin a este polvo. Mulelo cuidadosamente en la campana.) Agitando continuamente la pasta caf obtenida, agrega lentamente 16 mL de perxido de hidrgeno al 30% (PRECAUCIN!) con un embudo de separacin. Cuando la efervescencia haya terminado, agrega lentamente 60 mL de HCl concentrado. Contina la agitacin, con calentamiento a 85 C por 20 minutos; luego enfra la mezcla a temperatura ambiente y filtra el [Co(NH3)5Cl]Cl2 precipitado. Lava con 40 mL de agua helada, repartida en varias porciones, y posteriormente con 40 mL de HCl 6 M fro. Deja secar el producto en un horno a 100 C. El rendimiento esperado es de alrededor de 18 g de producto morado. El complejo puede ser recristalizado a partir de agua hirviente para dar pequeos cristales de color morado oscuro con buen rendimiento, pero esto no es necesario para la siguiente sntesis. b) Preparacin de [Co(NH3)5ONO]Cl2 y [Co(NH3)5NO2]Cl2. Disuelve 10 g de [Co(NH3)5Cl]Cl2 en una solucin de 15 mL de amoniaco concentrado (CUIDADO!) en 160 mL de agua, con agitacin y calentamiento constantes. Filtra cualquier precipitado de xido de cobalto que pudiera formarse, y enfra el filtrado a 10 C. Titula la solucin, enfriando constantemente, con HCl 2 M hasta que sea neutra al tornasol. La solucin cambiar a color vino. Disuelve 10.0 g de nitrito de sodio en la solucin, y posteriormente agrega 10 mL de HCl 6 M. Deja la solucin en un bao de hielo por una o dos horas. Filtra los cristales rosas de [Co(NH3)5ONO]Cl2. Lava con 50 mL de agua helada, seguidos por 50 mL de etanol, y deja secar a temperatura ambiente. El rendimiento esperado es de aproximadamente 9 g. Guarda el producto en un lugar fresco y oscuro. Con el tiempo, ocurre lentamente la isomerizacin del ismero nitrito al ismero nitro. Para preparar una muestra pura del ismero nitro, disuelve 4.0 g de [Co(NH3)5ONO]Cl2 en 40 mL de agua caliente que contenga algunas gotas de amoniaco acuoso. Enfra la solucin en bao de hielo y filtra el precipitado naranja de [Co(NH3)5NO2]Cl2. El rendimiento esperado es de alrededor de 3.5 g de producto amarillonaranja.

PROBLEMA 28 - EXPERIMENTAL
Separacin de nquel y cobalto por cromatografa de intercambio aninico, seguida por titulacin de los metales con EDTA por retroceso. El cobalto y el nquel pueden ser separados en una columna de intercambio aninico del tipo base fuerte (en forma de cloruros) eluyendo con HCl 9 M y HCl 3 M, respectivamente. En HCl 9 M, el nquel no forma un complejo cloro aninico

mientras que el cobalto s; por lo que el nquel ser eluido. En HCl 3 M, se disocia el complejo cloro aninico para formar el catin cobalto acuoso, el cual es eluido. Despus de la separacin, los metales son titulados indirectamente con EDTA estndar para cuantificarlos; se agrega un exceso de EDTA y el exceso se titula por retroceso con una solucin estndar de zinc en un medio ligeramente cido, usando naranja de xilenol como indicador. Separacin: Ni2+ + Cl- NiCl+ Co2+ + 4Cl- CoCl42Titulacin Co2+ + H2Y2- CoY2- + 2H+ Ni2+ + H2Y2- NiY2- + 2H+ H2Y2- + Zn2+ ZnY2- + 2H+ (titulacin por retroceso) Punto final H4In + Zn2+ ZnIn2- + 4H+
amarilloverde rojovioleta

Soluciones y reactivos: Miscelneos: NaOH 3 M, HCl 9 M, HCl 3 M, naranja de xilenol al 0.5% (w/v) en etanol al 10% (0.5 g disueltos en 10 mL de etanol y diluido hasta 100 mL con agua. La solucin debe estar fresca), grnulos de zinc, fenolftalena al 0.2% en etanol al 90%, hexamina (hexametilentetramina), resina de intercambio aninico Dowex 1-X8 (en forma de cloruros) o equivalente. Solucin de EDTA 0.01 M estndar: Preparar a partir de Na2H2Y2H2O secado en la estufa a 80 C durante dos horas y enfriado en un desecador. Pesa con exactitud aproximadamente 1.9 g, disuelve en agua destilada y desionizada, y diluye hasta 500.0 mL en un matraz volumtrico. Calcula la molaridad. Solucin de zinc 0.01 M estndar: Pesa con exactitud aproximadamente 0.33 g de zinc puro en grnulos y transfirelo a un vaso de precipitados de 400 mL. No uses polvo de zinc. Disuelve el zinc con la mnima cantidad necesaria de HCl, calentando en bao de vapor para acelerar la disolucin. Tapa el vaso con un vidrio de reloj durante la disolucin, para evitar salpicaduras. Una vez que el zinc est disuelto, lava las gotas de agua que estn en el vidrio de reloj con agua destilada, asegurndote de que el agua caiga dentro del vaso. Enjuaga tambin las paredes interiores del vaso y transfiere la solucin cuantitativamente a un matraz volumtrico de 500 mL. Afora con agua destilada. Calcula la molaridad de la solucin. Procedimiento

Para una exactitud mxima, los grnulos de zinc deberan ser tratados con HCl 2 M para remover cualquier capa de xido de zinc. Decanta el cido y lava el zinc repetidamente con agua. Luego lava varias veces con etanol y finalmente, en la campana de gases, con ter (PELIGRO DE INCENDIO). Seca los grnulos antes de pesarlos.

1.

Preparacin de la columna de intercambio inico. Prepara una columna usando una bureta de 50 mL, que tenga lana de vidrio en el fondo para sostener la resina. Agrega a la bureta una pasta de resina Dowex 1-X8 de intercambio aninico (en forma de cloruros) en HCl 9 M, hasta que la altura de la columna sea de 15 a 20 cm. No permitas en ningn momento que el nivel del lquido quede por debajo del nivel de la resina. Mantn unos 2 cm de lquido por arriba de la resina. Lava la columna con dos porciones de HCl 9 M, con un flujo de 2 o 3 mL por minuto. La columna tomar un color ms oscuro al ser tratada con el HCl, pero recuperar su color si se trata con agua. Deja 2 o 3 cm de HCl por arriba del nivel de la resina.

2.

Separacin de la muestra problema. Pdele a tu profesor una muestra problema en un matraz volumtrico de 50 mL. Llena hasta el aforo con HCl 9 M. La muestra deber contener 5 mmol o menos de nquel y de cobalto en HCl 9 M. Agrega con una pipeta 2 mL de la solucin problema a la columna. Eluye el nquel con aproximadamente 75 mL de HCl 9 M, agregado en porciones de 15 mL, con un flujo de 1 o 2 mL por minuto. Recibe el eluyente en un matraz Erlenmeyer de 250 mL. El complejo amarilloverde plido NiCl+ fluir por la columna y puede oscurecer la resina. La banda azul del complejo de cobalto (puede verse verde debido a que la resina es amarilla) recorrer parte de la columna. Mientras la muestra es eluida, es recomendable realizar una titulacin de prctica como la descrita en la seccin 3. Despus de que todo el nquel ha sido eluido pero antes de que la banda de cobalto llegue al final de la columna, detn el flujo y reemplaza el matraz receptor por uno limpio. Eluye el cobalto con unas cinco porciones de 10 mL de HCl 3 M, con un flujo de 1 mL por minuto. Al diluirse el HCl en la columna, el complejo azul CoCl 42- debe disociarse para dar el Co2+ de color rosa. Cuando pueda verse que ya ha sido eluido todo el cobalto, detn el flujo y procede a titular los metales por separado. (Puedes hacer una elucin final con agua para asegurarte de que todo el cobalto ha sido removido.)

3.

Titulacin del nquel y el cobalto. Evapora cuidadosamente las muestras recibidas, casi hasta sequedad en una parrilla caliente (EN LA CAMPANA). Enfra y diluye con 50 mL de agua destilada desionizada. Antes de titular tus muestras desconocidas, y mientras estn evaporndose, realiza una o dos titulaciones de prctica usando soluciones preparadas de nquel o cobalto. Titula las soluciones indirectamente de la siguiente manera: neutraliza cada solucin con NaOH 3 M usando fenolftalena como indicador, cuidando de no agregar NaOH en exceso. Agrega HCl gota a gota hasta que desaparezca el color rosa del indicador. Agrega 25.00 mL de EDTA estndar 0.01 M al matraz, 5 gotas de HCl 9 M, 1 g de hexamina, y cuatro gotas de la solucin indicadora de naranja de xilenol. La hexamina amortiguar el pH entre 5 y 6. Si la solucin es de color rojovioleta, calienta ligeramente y agrega otros 10.00 mL de EDTA. Titula por retroceso con la solucin estndar de zinc 0.01 M hasta que el indicador cambie de amarilloverde a rojovioleta.

Clculos A partir de los milimoles de EDTA usados y de los milimoles de EDTA en exceso encontrados en la titulacin por retroceso, calcula y reporta el nmero de milimoles de nquel y cobalto en tu muestra problema.

PROBLEMA 29 - EXPERIMENTAL
Determinacin de cobre y bario en una mezcla de cloruros. El cobre y el bario son componentes comunes en los superconductores de alta temperatura. Pueden ser separados fcilmente removiendo el bario como su sulfato insoluble, el cual se determina gravimtricamente. El cobre soluble puede ser determinado volumtricamente por medio de una reaccin con yoduro en exceso, seguida por una determinacin del yodo liberado con tiosulfato estandarizado. Separacin Ba2++SO42-BaSO4 Titulacin 2Cu2++ 4I-2CuI+I2 I2 + 2S2O32- 2I- + S4O62Soluciones y reactivos Miscelneos: Solucin de KI al 10%, KSCN o NH 4SCN al 10%, indicador de almidn, K2SO4 saturado. Na2S2O3 0.01 M estndar: Esta solucin se prepara disolviendo 25.0 g de Na2S2O35H2O R.A. en 1 L de agua destilada y hervida. Es recomendable usar soluciones frescas o estabilizadas. Estandariza tu solucin de tiosulfato de sodio usando yodato de potasio o bromato de potasio como patrn primario. Este procedimiento puede encontrarse en cualquier texto sobre anlisis cuantitativo o anlisis volumtrico. Procedimiento 1. Pesa con exactitud aproximadamente 3 g de tu muestra problema (la muestra problema debe tener CuCl2 y BaCl2 en proporcin entre 3:1 y 8:3). En un vaso de precipitados de 400 mL, disuelve el slido en 180 mL de agua. (No uses ms agua, ya que el volumen final ser de 250 mL ver la parte final del paso 3.) Precipitacin. Calienta la solucin casi hasta hervir y mantnla justo por debajo del punto de ebullicin. Agrega lentamente con una pipeta Pasteur 10 mL solucin saturada de K2SO4, agitando vigorosamente durante la adicin. Permite que se asiente el precipitado, y asegrate de que la precipitacin ha concluido agregando algunas gotas de la solucin de sulfato de potasio, sin agitar. Si se forma ms precipitado, agrega lentamente, agitando, 5 mL ms de sulfato de potasio; deja que se asiente y repite la prueba. Repite la operacin hasta que la precipitacin sea completa. Deja la varilla agitadora en el vaso, cbrelo con un vidrio de reloj y djalo digerir en un bao

2.

de vapor hasta que el lquido sobrenadante est transparente. (El precipitado inicial es de partculas muy finas. Durante la digestin, las partculas crecen hasta alcanzar un tamao filtrable.) Esto tomar de 30 a 60 minutos o ms. Agrega ms agua destilada si el volumen baja de 150 mL. 3. Filtracin y lavado del precipitado. Prepara para la filtracin un papel filtro Whatman de 11 cm del No. 42 (endurecido y sin cenizas) o equivalente. El papel debe estar bien ajustado al embudo para que el tallo largo del embudo se mantenga lleno de agua destilada, de lo contrario la filtracin ser muy lenta. Filtra la solucin caliente, con cuidado de no llenar demasiado el cono de papel, ya que el sulfato de bario tiene una tendencia a trepar por el papel. Lava el precipitado en el filtro con agua destilada caliente, limpia el precipitado de la varilla de vidrio y las paredes del vaso usando el gendarme de hule, y enjuaga nuevamente el vaso, transfiriendo su contenido al embudo. Examina el vaso muy cuidadosamente para asegurarte de que no haya partculas de precipitado que no hayan sido transferidas. Lava el precipitado y el papel con agua destilada caliente hasta que no se detecten cloruros en el agua de lavado* . Durante el lavado, intenta que el precipitado se vaya todo al fondo del cono de papel. Examina el filtrado para asegurarte de que no pas nada de precipitado a travs del filtro. Cuando ya hayas lavado minuciosamente el precipitado, prueba una gota del agua de lavado con amoniaco concentrado (CUIDADO!). Si no se pone de color azul es porque ya ha sido removido todo el cobre del precipitado. Si aparece un color azul, debes continuar lavando y repetir la prueba. Transfiere cuantitativamente el lquido filtrado a un matraz volumtrico de 250 mL y llena hasta el aforo. Mezcla completamente. 4. Titulacin. Pasa 50.0 mL de esta solucin a un matraz Erlenmeyer, usando una pipeta volumtrica; agrega 2 gotas de cido actico glacial (CUIDADO!), y 15 mL de KI al 10%. Titula el yodo liberado con la solucin 0.1 M de tiosulfato de sodio hasta que empiece a desvanecerse el color caf. Agrega 2 mL de indicador de almidn y contina la titulacin hasta que empiece a desaparecer el color azul. Agrega entonces ~ 10 mL de tiocianato de potasio (o de amonio) al 10%. Esto intensifica el color azul. Una vez agregado el KSCN, termina la titulacin lo ms rpidamente posible. Cuando el color azul haya desaparecido, deber quedar un precipitado de color plido. Debe obtenerse fcilmente un punto final bien definido. Repite con otras dos alcuotas o hasta que el resultado sea reproducible. Si lo deseas, puedes determinar el bario por anlisis gravimtrico de la siguiente manera: 5. Ignicin y pesado del precipitado. Afloja el papel en el embudo y djalo drenar por algunos minutos. Dobla el papel en forma de un paquete que contenga al precipitado, con el grosor triple del papel hacia arriba. Colcalo en el fondo de un crisol de porcelana previamente pesado. Revisa el embudo para asegurarte de que no queden trazas de precipitado; si las hay, lmpialas con un pedazo de papel de filtro y colcalo

Para realizar la prueba de cloruros, toma algunos mililitros del agua de lavado, acidifcalos con cido ntrico (CUIDADO!), y agrega unas gotas de solucin de nitrato de plata. Si aparece una turbidez es porque todava hay cloruros en el agua de lavado.

en el crisol. Coloca el crisol en un tringulo de porcelana en un anillo o tripi, en una posicin inclinada y con la tapa ligeramente abierta. Calienta suavemente con un mechero (CUIDADO!), hasta que haya sido eliminada toda la humedad y el papel comience a carbonizarse. Ajusta la llama de tal modo que el papel se carbonice sin inflamarse. Si se enciende el papel, tapa el crisol para apagar el fuego, y disminuye la intensidad de la flama. Cuando el papel est totalmente carbonizado y no desprenda ms humo, aumenta la temperatura gradualmente para quemar el carbn completamente. Es normal que el carbn produzca un brillo rojo al quemarse, pero no debe haber llamas. El precipitado debe ser blanco y sin partculas negras. Permite que se enfre a temperatura ambiente. Humedece el precipitado con tres o cuatro gotas de cido sulfrico diluido (1:4). Calienta lentamente hasta que todo el cido se haya evaporado. (Este tratamiento sirve para convertir a sulfato de bario cualquier parte del precipitado que pudiera haber sido reducida a sulfuro de bario por el carbn y el calentamiento.) Tapa el crisol y calintalo al rojo en la llama intensa del mechero por 15 minutos. Deja que se enfre el crisol en un desecador por lo menos durante una hora y psalo. Calintalo de nuevo por 1015 minutos, enfralo en el desecador y psalo de nuevo. Repite la operacin hasta que obtengas dos pesadas consecutivas que no difieran en ms de 0.4 mg. Clculos Cobre. A partir del volumen de tiosulfato utilizado, calcula los milimoles de yodo, y por lo tanto de cobre, presentes en la alcuota de 50.0 mL. Como la alcuota fue tomada de un total de 250.0 mL, los milimoles totales de cobre sern 5 veces los encontrados en la titulacin. Bario. Los milimoles totales de bario se obtienen directamente de la masa de sulfato de bario. Reporta en tus resultados la relacin Cu:Ba.

PROBLEMA 30 - EXPERIMENTAL
Preparacin y anlisis del tris(oxalato)ferrato (III) de potasio trihidratado, K3[Fe(C2O4)3]3H2O Introduccin En este experimento preparars el anin complejo tris(oxalato)ferrato (III), [Fe(C2O4)3]3-, y lo aislars como su sal de potasio trihidratada, K3[Fe(C2O4)3]3H2O. Este trihidrato es una sal cristalina verde, soluble en agua caliente pero bastante insoluble en agua fra. El anin complejo es fotosensible. Esto implica que cuando se expone a luz de la longitud de onda apropiada (< 450 nm en este caso), el [Fe(C2O4)3]3- sufre una reaccin redox intramolecular en la que el tomo de Fe(III) se reduce a Fe(II), mientras que uno de los grupos oxalato se oxida a CO 2. Realizars esta fotorreduccin en tu muestra, y aplicars la reaccin fotoqumica para hacer una copia heliogrfica.

Finalmente, analizars el producto para determinar su contenido de agua, oxalato y hierro. ADVERTENCIA: El ion oxalato es txico y en el procedimiento de anlisis para el hierro se utiliza zinc que ha sido amalgamado con mercurio. Asegrate de desechar los residuos que contengan oxalato o mercurio de manera responsable! Preparacin del K3[Fe(C2O4)3]3H2O Pesa aproximadamente 9.0 g de oxalato de potasio hidratado (K 2C2O4H2O) en un vaso de precipitados de 250 mL, agrega 30 mL de agua destilada y calienta para disolver (sin hervir). En un vaso de precipitados pequeo, disuelve 4.4 g de FeCl36H2O en la mnima cantidad posible de agua fra (1015 mL). Agrega la solucin de FeCl 36H2O a la solucin caliente de oxalato y revuelve con una varilla de vidrio. Permite que el producto cristalice (en un lugar donde no est expuesto al sol o a otra fuente de luz intensa) enfriando la solucin en un bao de hielo. Separa el producto cristalino por filtracin al vaco (descarta el filtrado en una botella de desechos apropiada). Lava los cristales con unos 10 mL de acetona y contina la succin por unos minutos hasta que el producto quede seco. Recristalizacin: Pesa 5.0 g de tu K 3[Fe(C2O4)3]3H2O en un vaso de precipitados pequeo. Agrega 13 mL de agua destilada y calienta la mezcla en una parrilla hasta que el slido se disuelva. Filtra la solucin rpidamente por filtracin al vaco, lava el filtro con no ms de 2 mL de agua destilada caliente, y transfiere inmediatamente el filtrado caliente a un vaso de precipitados limpio. Enfra la solucin en bao de hielo y, cuando la cristalizacin haya terminado, filtra los cristales y lvalos con dos volmenes de 5 mL de agua destilada helada. Seca los cristales por succin. Transfirelos a un recipiente previamente pesado y scalos a 50 C por unos 30 minutos. Pesa el producto purificado. Reaccin fotoqumica del [Fe(C2O4)3]3Como mencionamos en la introduccin, la luz provoca que ocurra una reaccin de transferencia de electrones interna en el ion [Fe(C 2O4)3]3-, produciendo CO2 e iones Fe2+. El Fe2+ que se produce puede ser detectado fcilmente agregando una solucin de ferricianuro de potasio, K3[Fe(CN)6], 0.1 M, con lo cual debe producirse un color azul profundo debido a la formacin del complejo ferroferricianuro (azul de Turnbull). Para este ejercicio necesitars una fuente de luz intensa, tal como un reflector de 150 W (CUIDADO!). La luz del sol puede ser suficiente, pero la reaccin fotoqumica ser bastante ms lenta. Debes tener CUIDADO al manejar la solucin de ferricianuro. Evita el contacto con la piel y junta todos los residuos para que puedan ser tratados adecuadamente. Lvate bien las manos despus de manipularlo. Disuelve 0.7 g de tu complejo (no necesita ser recristalizado) en 100 mL de agua destilada en un matraz Erlenmeyer. Agrega 3 mL de H2SO4 2 M y revuelve la mezcla. Etiqueta tres tubos de ensaye y en cada uno coloca 10 mL de esta solucin.

Guarda uno de los tubos en un lugar oscuro como control e irradia los otros dos tubos con la fuente de luz durante 1 y 5 minutos, respectivamente. A cada tubo agrgale 1 mL de la solucin de ferricianuro de potasio (K3[Fe(CN)6]). Registra tus observaciones y explcalas. Para convencerte de que es Fe 2+ el que ests detectando, agrega unas gotas de la solucin de ferricianuro a un tubo que contenga 5 mL de solucin de sulfato de hierro(II), FeSO4. Copia heliogrfica Coloca 25 mL de la solucin de [Fe(C 2O4)3]3- en una caja de Petri. Empapa un pedazo de papel de filtro de 5 cm 5 cm con esta solucin. Usando pinzas de plstico, saca el papel y djalo secar en la campana con la luz apagada. Cuando el papel est seco, colcalo aproximadamente 75 mm debajo de la fuente luminosa y coloca algunos objetos opacos (llaves, monedas, etc.) sobre el papel. Irradia al papel por unos minutos y luego remjalo en otra caja de Petri que contenga unos 25 mL de solucin de ferricianuro de potasio (CUIDADO!) usando pinzas de plstico. Remueve la copia heliogrfica revelada y enjugala en un vaso de precipitados que contenga agua destilada, para eliminar el exceso de ferricianuro. Explica tus observaciones. Anlisis del complejo Preparacin de la solucin de KMnO4 0.04 M: Disuelve 6.3 g de KMnO4 R.A. en 1 L de agua destilada. Si la solucin no va a usarse inmediatamente, hirvela suavemente por una hora. Tpala y djala reposar toda la noche. Elimina cualquier precipitado de MnO2 filtrando la solucin con un embudo de vidrio poroso. La solucin deber guardarse en un lugar oscuro cuando no se use. Estandarizacin de la solucin de KMnO4 0.04 M: Pesa con exactitud aproximadamente 3.2 g de polvo de oxalato de sodio R.A. (Na2C2O4) en un vaso de 250 mL. Disuelve el oxalato de sodio usando la cantidad mnima de agua destilada caliente (aproximadamente 100 mL). Transfiere la solucin cuantitativamente a un matraz volumtrico de 250 mL, y llena hasta la lnea de aforo con agua destilada. Pipetea 25.0 mL de esta solucin a un matraz Erlenmeyer de 250 mL limpio, y agrega 25.0 mL de H2SO4 1 M. Calienta la solucin a 60 C (como gua aproximada, 60 C es la temperatura a la cual el matraz est apenas demasiado caliente para tenerlo cmodamente en la mano [CUIDADO!]). Titula la solucin caliente con el KMnO 4 0.04 M hasta que persista un ligero color rosa durante ms de 30 segundos. Repite la determinacin hasta que tres titulaciones seguidas no difieran en ms de 0.1 mL. Promedia tus resultados y calcula la molaridad de la solucin de KMnO4. Determinacin de agua: Limpia y seca una pequea caja de Petri o vidrio de reloj. Coloca el plato en un horno de microondas por alrededor de un minuto. ADVERTENCIA: El horno de microondas debe contener balasto para evitar daarlo. Por ejemplo, puede usarse un vaso lleno de gel de slice o arena. Un horno convencional, a 110 C, funciona tambin solamente tarda ms el proceso de secado.

Pesa con exactitud aproximadamente 1.0 g del complejo recristalizado y colcalo en la caja de Petri. Calintalo en el horno de microondas por 5 minutos en high. Deja enfriar y pesa. Repite la operacin hasta que la muestra alcance un peso constante. La prdida de peso es debida a la eliminacin del agua de cristalizacin. Calcula el porcentaje de agua de cristalizacin en tu muestra. Determinacin de oxalato y de hierro Los ligantes oxalato en el tris(oxalato)ferrato(III) se oxidan por titulacin con permanganato: 5[Fe(C2O4)3]3- + 6MnO4- 48H+ 5Fe3+ + 6Mn2+ + 30CO2 + 24H2O El hierro(III) se reduce a hierro(II) usando un exceso de amalgama de zinc (zinc metlico recubierto con mercurio metlico). 2Fe3+ + Zn(Hg) 2Fe2+ + Zn2+ + Hg El hierro(II) formado es reoxidado cuantitativamente a hierro(III) titulndolo con la solucin de permanganato: 5Fe2+ + MnO4- + 8H+ 5Fe3+ + Mn2+ + 4H2O Observa que cualquier impureza de hierro(II) en tu muestra tambin sera titulada en este paso. En resumen, en una muestra pesada de tu complejo: 1. se determina el oxalato por titulacin con permanganato, 2. el Fe(III) presente en la solucin resultante se reduce a Fe(II) usando un exceso de amalgama de zinc, y 3. El Fe(II) se determina por titulacin con permanganato. Procedimiento Pesa con exactitud aproximadamente 0.40 g del complejo recristalizado y disuelve la muestra en 25 mL de cido sulfrico 1 M en un matraz Erlenmeyer de 125 mL. Calienta la solucin a 60 C y determina la cantidad de oxalato en la solucin con el KMnO4 como se explic anteriormente para la estandarizacin. No te pases del punto final! Enfra la solucin y transfirela cuantitativamente en una botella que contenga la amalgama de zinc. Agita la botella por dos minutos y decanta cuidadosamente la solucin reducida a un matraz Erlenmeyer de 250 mL. Lava minuciosamente la amalgama agitndola con tres porciones de 30 mL de H2SO4 ~ 0.5 M, agregando el lquido de enjuague al matraz. Determina el contenido de Fe(II) en la solucin titulndolo con KMnO4 a temperatura ambiente. Calcula los porcentajes de hierro y oxalato en tu muestra.

Respuestas a los problemas


PROBLEMA 1
a)
H C C H C C H H H H H C C C C H

Ciclobutadieno b)

Butatrieno

La hibridacin de cada tomo es como se muestra a continuacin:


H

sp2
C C

sp
C C

sp

sp

El enlace central C=C est formado por dos tomos con hibridacin sp, mientras que los otros enlaces C=C se encuentran entre un tomo con hibridacin sp y un tomo con hibridacin sp2. Cuanto mayor sea la proporcin de carcter s en el orbital hbrido, ms redondo ser el orbital, y por lo tanto ser ms corto el enlace resultante. Entonces, el enlace central C=C es ms corto que los otros dos. c) H C H C C H d) i) ii) iii) i) Uno. Uno. Uno. Puede decirse que tres:
Cl C C Cl C C H H H Cl C C C C H H Cl Cl C C C C Cl H

C H

Tetraedrano

e)

II

III

Como el ciclobutadieno no es aromtico, I y II no son propiamente formas cannicas diferentes sino ismeros distintos (sin embargo, no pueden existir por separado).

ii)
Cl

Tres:
Cl C H C C C H H Cl C C C C Cl Cl H Cl C C C C H H

I iii) Uno:

II

III

Cl C

C Cl

f)

Los puntos de fusin de los ismeros deben estar relacionados con sus momentos dipolares. Por simetra, II tiene momento dipolar de cero y por lo tanto el punto de fusin ms bajo. De los otros dos, III tiene un momento dipolar mayor que el de I (debido a que el dipolo en III queda a lo largo del eje horizontal largo, mientras que el dipolo en III queda a lo largo del eje vertical corto). Entonces III tendr el punto de fusin ms alto.

PROBLEMA 2
a)

Energa 1s (= 1) 1s (= 2) H2 H

H b)

Los orbitales moleculares del H2 se forman combinando, en igual proporcin, los orbitales atmicos 1s de cada tomo de hidrgeno. Se forman dos orbitales moleculares: uno que tiene mayor energa que los orbitales atmicos 1 s, y uno con menor energa. El orbital molecular (MO) de menor energa se forma por la combinacin en fase de los orbitales 1s de los tomos de hidrgeno. El MO de mayor energa se forma por la combinacin fuera de fase. De acuerdo con la figura anterior, podemos escribir estas combinaciones de la siguiente manera: = 1 + 2 y * = 1 - 2

c)

Se le llama orbital molecular de antienlace porque produce un decremento neto en la densidad electrnica entre los tomos.

d)
2pz i j

2py

h g z

2px

f e x y

2s

d c

1s

b a

O2

e)

El traslape lineal de los dos orbitales p en g es mucho mejor que el traslape lateral de los orbitales en e o en i. Por esto el carcter enlazante en g es mayor y por lo tanto su energa es menor. Una razn similar explica la diferencia en la energa de h vs la de f y j. El traslape en h es mejor que en f y j. Entonces el carcter antienlazante en h es mayor que en f y j, por lo que su energa es mayor.

f)

Los orbitales e e i surgen del traslape lado a lado de los orbitales atmicos p. Los orbitales p en la direccin x son, por supuesto, idnticos a los que se encuentran en la direccin y (x e y son asignados arbitrariamente por nosotros) por lo que los orbitales moleculares resultantes son tambin idnticos. Al estirar la molcula de O2 el traslape entre los orbitales atmicos disminuye. Por esto disminuye el carcter de antienlace en el orbital j, y por lo tanto la energa disminuye. Como el traslape lineal en h es mayor inicialmente que el traslape lateral en j, el traslape disminuir ms rpidamente en h al estirar la molcula de O2. Entonces el carcter de antienlace disminuir ms rpidamente, y por lo tanto la energa de h disminuir ms rpidamente.

g)

PROBLEMA 3
a) Podemos construir los siguientes diagramas de orbitales moleculares:
2 px , 2py , 2pz 2s 2s 2 px , 2py , 2pz

1s 1s

2 px , 2py , 2pz 2s 2s 2 px , 2py , 2pz

1s 1s

2 px , 2py , 2pz 2s 2s 2 px , 2py , 2pz

1s 1s

En estos diagramas no se considera la diferencia entre los niveles de energa de los orbitales de tomos diferentes, pero se muestran las tendencias cualitativas correctas para la determinacin de los rdenes de enlace. Los orbitales moleculares resultantes de los orbitales atmicos 1s y 2s quedan llenos en todos los casos, por lo que no dan lugar a ningn carcter de enlace neto. El orden de enlace se obtiene de la diferencia entre el nmero de orbitales de enlace y de antienlace ocupados, que surgen de los orbitales atmicos 2 p. El CN tiene cinco electrones en los orbitales de enlace y que surgen de los orbitales 2p y ninguno en los orbitales de antienlace, por lo que tiene un orden de enlace de 2.5. El N 2 tiene seis electrones en los orbitales de enlace, por lo que tiene un orden de enlace de 3. El NO

tiene una diferencia de cinco electrones entre los orbitales de enlace y los de antienlace, por lo que el orden de enlace es de 2.5. b) Aqu tenemos que considerar la configuracin electrnica que resulta de la prdida de un electrn. Para CN+, la nueva configuracin ser:
2 px , 2py , 2pz 2s 2s 2 px , 2py , 2pz

1s 1s

Para el N2+, se remueve un electrn de uno de los orbitales de enlace del N2, mientras que para el NO+, se pierde el electrn del orbital * de antienlace del NO. La prdida de un electrn le dar cierta estabilidad al NO+, por lo que el NO tendr la energa de ionizacin ms pequea. La ionizacin del CN y del N 2 implica remover un electrn de un par de electrones en un orbital de enlace, por lo que EI(CN) debe ser similar a EI(N2). Sin embargo, puede esperarse que EI(N 2) sea algo mayor a EI(CN) debido a que el traslape entre los orbitales atmicos de los dos tomos de nitrgeno debe ser mejor que el traslape entre los orbitales de un tomo de nitrgeno y uno de carbono. Los valores reportados estn de acuerdo con estas predicciones: EI(CN) = 1359 kJ mol-1, EI(N2) = 1503 kJ mol-1, y EI(NO) = 894 kJ mol-1. Ntese que EI(NO) es mucho menor que las otras dos, indicando que es mucho ms fcil remover un electrn de un orbital de antienlace que de uno de enlace. c) La formacin del N2- y del NO- implica, en ambos casos, la adicin de un electrn a un orbital de antienlace. En cambio, la formacin del CN - implica colocar el electrn adicional en el orbital de enlace (y adems se obtiene una configuracin que es isoelectrnica con la del N2). Por esto esperamos que el CN tenga la mayor afinidad electrnica, lo cual concuerda tambin con los valores reportados: AE(CN) = 369 kJ mol-1, AE(N2) ~ 0 kJ mol-1, y AE(NO) ~ 9 kJ mol-1. d) Aqu se tienen dos efectos en competencia. Primero, el traslape ser generalmente mayor entre orbitales atmicos de tomos iguales, por lo que esperaramos que el N 2 tuviera la fuerza de enlace mayor. Sin embargo, la comparacin resulta ms complicada debido a que el NO+ y el CN- son especies cargadas. Los procesos de disociacin son, respectivamente: CN C- + N

(debido a que el carbono tiene una afinidad electrnica mayor a la del nitrgeno) N2 NO+ N + N N + O+

(debido a que el oxgeno tiene una energa de ionizacin menor a la del nitrgeno).

La formacin de enlaces tiende a estabilizar las cargas, sean positivas o negativas, por lo que el mejor traslape en el N 2 no implica necesariamente que sea el N 2 el que tenga la mayor fuerza de enlace de estas tres especies isoelectrnicas. A falta de mayor informacin, la pregunta no puede ser contestada con certeza. (Como dato cultural, los valores reportados son: D[(CN)-] = 994 kJ mol-1; D(NN) = 946 kJ mol-1; y D[(NO)+] = 1051 kJ mol-1. Entonces la deslocalizacin de la carga predomin sobre el mejor traslape NN en ambos casos.)

PROBLEMA 4
a)

F Xe F

F F Xe F F

El XeF2 tiene 5 pares de electrones en el Xe, por lo que su estructura estar basada en una configuracin trigonal bipiramidal. De las tres posibilidades,
F F Xe F F Xe F Xe F

la estructura lineal minimiza las repulsiones entre los pares no compartidos (que se mantienen ms cerca del Xe que los tomos que forman parte de los enlaces XeF) por lo que se favorece la geometra lineal. El XeF4 tiene 6 pares de electrones en el Xe, por lo que la estructura estar basada en una configuracin octadrica. De las dos posibilidades,
F F Xe F F F F Xe F F

la estructura plana minimiza las repulsiones entre los pares no compartidos, y es por lo tanto la preferida. b) El flor siempre tiene un nmero de oxidacin de -1. Por lo tanto los nmeros de oxidacin del xenn son de +2 (en el XeF 2) y +4 (en el XeF 4). Estas especies son agentes oxidantes potentes! Ignorando las diferencias entre los niveles energticos del H y del He, podemos dibujar los siguientes diagramas de orbitales moleculares: 1s He H+ 1s

c)

1s He 1s He He 2+ 1s He + 1s

En estos diagramas podemos ver que el HeH+ y el He22+ tienen ambos un orden de enlace de 1, mientras que el He2+ tiene un orden de enlace de . d) Los elementos del grupo 2 tienen segundas energas de ionizacin bajas (porque Be+ Be2+ o Mg+ Mg2+ dan una capa cerrada, la configuracin de gas noble 1 s2, o 1s2 2s2 2p6). El Mg2+ tiene un efecto pantalla mayor que el del Be 2+, por lo que EI(Mg+) < EI(Be+). Por lo tanto el magnesio es el mejor candidato para Z. e) De los vecinos del magnesio, el calcio tiene la menor segunda energa de ionizacin del conjunto {Be, Na, Al, Ca} por razones similares a las expuestas anteriormente. Entonces el calcio es el que tiene ms posibilidades de formar un dicatin estable con el helio. El ion Na+ ya tiene una capa cerrada, pro lo que la ionizacin Na + Na2+ es muy desfavorable. Entonces el sodio es el que tiene la menor probabilidad de formar un dicatin con el helio.

PROBLEMA 5
a) b)
H H H H H L L H H Fenantreno H C L H H H H H H L H H Trifenileno Pireno H H L L H H H H H H H H L H L L L H H C H L H H

Las estructuras se muestran en b), a continuacin.

c)

El fenantreno tiene dos anillos aromticos de tres (67%). El trifenileno tiene tres anillos aromticos de cuatro (75%). El pireno tiene dos anillos aromticos de cuatro (50%). Por lo tanto, el trifenileno es el ms aromtico de estos PAHs, y el pireno el menos aromtico. El fenantreno tiene 2 anillos aromticos para 14 tomos de carbono (1/7). El trifenileno tiene 3 anillos aromticos para 18 tomos de carbono (1/6). El pireno tiene 2 anillos aromticos para 16 tomos de carbono (1/8). El trifenileno sigue siendo el ms aromtico de estos PAHs, y el pireno el menos aromtico.

d)

e)

i)

Orden de enlace promedio = 4/3. En cualquier forma cannica, uno de cada tres enlaces CC debe ser un doble enlace, y dos deben ser sencillos. Alternativamente (ver la estructura de resonancia que se muestra a continuacin) dos de cada tres enlaces CC tienen un orden de enlace de 3/2, mientras que el tercero es formalmente sencillo: nuevamente se obtiene un orden de enlace promedio de 4/3.

ii)

El orden de enlace promedio en el benceno es de 3/2. Los enlaces CC en el benceno tienen un carcter de doble enlace mayor que los del grafito. Por lo tanto, los enlaces en el grafito son ms largos. Slo uno de cada tres anillos en el grafito es aromtico en cualquier forma cannica dada. Sin embargo, hay un anillo por cada dos tomos de carbono en el grafito, mientras que en el benceno hay un anillo por cada seis tomos de carbono. Segn este criterio, el benceno y el grafito son igualmente aromticos. (Si se consideraran sistemas de deslocalizacin ms grandes, como por ejemplo sistemas de 10 electrones, el grafito resultara ms aromtico que el benceno ya que estos sistemas evidentemente no pueden presentarse en las estructuras resonantes del benceno.)

iii)

PROBLEMA 6
Los seis ismeros C4H8 son el 1-buteno, cis-2-buteno, trans-2-buteno, metilpropeno, ciclobutano, y metilciclopropano. A, B, C y D decoloran al bromo en ausencia de luz. Este es el proceso de adicin de bromo a un alqueno. Por lo tanto, AD son los cuatro alquenos; E y F son los dos cicloalcanos. El metilciclopropano tiene un momento dipolar, mientras que el momento dipolar del ciclobutano es de cero. Por lo tanto, el punto de ebullicin del metilciclopropano ser mayor que el del ciclobutano. Como T eb(E) > Teb(F), E es el metilciclopropano y F es el ciclobutano. La hidrogenacin del 1-buteno, o de cualquiera de los ismeros del 2-buteno, da n-butano. Esto explica el hecho de que la hidrogenacin de A, B y C d un mismo producto. El que falta, D, es el metilpropeno. La adicin de bromo al cis-2-buteno da la forma meso del 2,3-dibromopropano, mientras que la adicin al trans-2-buteno da los enantimeros R, R y S, S. Esto explica la observacin de que B y C produzcan productos estereoisomricos con el

bromo. Por eliminacin (el proceso deductivo, NO el mecanismo orgnico!), A es el 1-buteno. El cis-2-buteno debe tener un punto de ebullicin mayor que el del trans-2-buteno, debido a que este ltimo carece de momento dipolar. Como Teb( C) > Teb(B), C es el cis-2-buteno y B es el trans-2-buteno.

PROBLEMA 7
a) Tres ismeros:
Cl C H C H H Cl Cl C C Cl Cl H Cl C C H H

I I y II son ismeros geomtricos. b) Cl C H I I y II son ismeros pticos. c) C C Cl H H Cl C

II

III

Cl C II C H

Cl C Cl III C C

H H

Se tienen ismeros geomtricos cuando n es par. Cuando n es non, se tienen ismeros pticos. Esta diferencia se debe a la planaridad de las estructuras con n par, que evidentemente deben tener un plano de simetra, y por lo tanto todas pueden ser superpuestas en sus imgenes especulares. La estructuras con n non no son planas debido a que los dos enlaces C=C terminales tienen nubes electrnicas que son ortogonales entre s. Estas estructuras no poseen planos de simetra y por lo tanto pueden tener imgenes especulares no superponibles (esta es la verdadera base de la isomera ptica).

PROBLEMA 8
a) El ion en m/z = 82 debe se 12C35Cl35Cl. La ausencia de seal en m/z = 83 implica que no hay 13C en la muestra. El ion en m/z = 28 debe ser entonces 12C12C2H2H. La ausencia de seal en m/z = 27 implica la ausencia de 1H en la muestra. Por lo tanto, el ion en m/z = 75 debe ser 35Cl12C32H2, y la de m/z = 77 debe ser el isotopmero que contiene 37Cl. b) I(m/z = 77) = 0.6 I(m/z = 75) n(37Cl) = 0.6 n(35Cl) %(37Cl) = [0.6/(1+0.6)] 100% = 37.5% %(35Cl) = 62.5% Hay tres isotopmeros del dicloropropadieno:

%[C3D2(35Cl)2] = (63.5%)2 = 39.06% %[C3D235Cl37Cl] = 2 62.5% 37.5% = 46.88% %[C3D2(37Cl)2] = (37.5%)2 = 14.06% c) 1 mol corresponde a: 0.3906 mol 110.0 g mol-1 + 0.4688 mol 112.0 g mol-1 + 0.1406 mol 114.0 g mol-1, lo cual da una masa molar de 111.50 g mol-1. d) Los cationes CCl2+ y CCD2+ slo pueden surgir si la estructura del dicloropropadieno es la siguiente: La ruptura de este enlace produce ambos iones Cl C Cl e) C C D D

El pico en m/z = 242 en el modo de deteccin de iones positivos se debe a la especie (C4H9)4N+ (C16H36N = 242.4 uma), la cual es la nica especie cargada positivamente en la solucin. En el modo de deteccin de iones negativos, se observan los picos debidos al 79Br- y al 81Br-. Las abundancias naturales del 79Br y del 81Br son de 50.7% y de 49.3%, respectivamente, por lo que estos dos picos tienen prcticamente la misma altura. Estos espectros concuerdan con la autoionizacin del isopropanol: C3H7OH C3H7Om/z = 59.1

f)

C3H7OH2+
m/z = 61.1

PROBLEMA 9
Podemos construir un ciclo termoqumico donde las cantidades conocidas son: A = 2 Hf(Li(g)) = 318.8 kJ mol-1 B = EI(Li(g)) = 520.3 kJ mol-1 C = -D(Li2+(g)) = -129.8 kJ mol-1 D = -EI(Li2(g)) = -493.3 kJ mol-1 Li+ (g) + Li C
(g)

Li2+ (g ) D

B Li (g) + Li F
(g)

Li2 (g )

A 2Li (s) y las incgnitas son:

E = A + B + C + D = Hf(Li2(g)) y F = A - E = D(Li2(g)) De este modo obtenemos Hf(Li2(g)) = 216.0 kJ mol-1 y D(Li2(g)) = 102.8 kJ mol-1. b) El nmero de moles de Li en la muestra es (122.045 g / 6.9410 g mol-1) = 17.583 mol. Podemos calcular la presin que se esperara si slo hubiera Li (g) presente, usando la ley de los gases ideales, PV = nRT: P = nRT / V es decir, P = 17.483 mol 8.31441 J K-1 mol-1 610.25 K / (5.9474 105 m3), dando Pesperada = 0.15000 Pa (o 1.1251 10-3 Torr). Ahora, tomando en cuenta el equilibrio Li(g) + Li(g) Li2(g),

podemos ver que dos moles de Li(g) producen 1 mol de Li2(g), por lo que la presin parcial de dilitio debe ser la diferencia entre la presin esperada para el litio atmico y la presin real observada. P(Li2(g)) = 1.1251 10-3 Torr - 9.462 10-4 Torr = 1.789 10-4 Torr. De igual manera, P(Li(g)) = Ptotal - P(Li2(g)) = 9.462 10-4 Torr - 1.789 10-4 Torr, por lo que P(Li(g)) = 7.673 10-4 Torr. Para determinar Kc, debemos convertir las presiones parciales en concentraciones. A partir de PV = nRT y los factores de conversin correspondientes, encontramos que a 610.25 K, 1 Torr equivale a 2.6276 10-5 mol L-1, con lo que obtenemos [Li 2(g)] = 4.701 10-9 mol L-1 y [Li(g)] = 2.016 10-8 mol L-1. Ahora, ya que Kc se define como
Kc =

[ Li ] [ Li ]
2(g) (g)

obtenemos un valor de Kc = 1.156 107. c) En esta cmara de reaccin, moles de Li = 265.384 g / 6.9410 g mol -1 = 38.234 mol. Podemos comprobar que no todo el litio esta el fase vapor: Si todo el Li(s) se convirtiera a Li2(g), entonces habra 38.234 mol en un volumen de 5.9474 108 L, lo que correspondera a [Li2(g)] = 3.2143 10-8 mol L-1, o una presin total de 1.2233 10-3 Torr. Esta presin es mayor que la observada en la prctica; adems, como gran parte del vapor estar formado por litio monoatmico en lugar de diatmico, la presin total debera ser bastante mayor que 1.2233 10-3 Torr, si todo el litio se evaporara. Podemos concluir que como la presin ya se estabiliz, el vapor se encuentra en equilibrio con litio slido o lquido. Entonces la presin observada corresponde a la presin de vapor del litio. La presin de vapor del dilitio puede definirse como la presin parcial de Li2(g) en la fase gaseosa bajo estas condiciones. Podemos resolver esto a partir de Kc, sustituyendo [Li(g)] + [Li2(g)] = 2.7472 10-8 L mol-1 (calculado a partir de la presin observada, usando el factor de conversin Torr

mol L-1 encontrado en b)), con lo cual encontramos que [Li 2(g)] = 5.553 10-9 mol L-1, lo cual corresponde a Pvap(Li2(g)) = 2.113 10-4 Torr a 610.25 K. d) Primero debemos determinar la energa necesaria para acelerar al Enterprise a la mitad de la velocidad de la luz. La energa cintica es E = mv2, y como m = 3.586 106 kg y v = 1.49896 108 m s-1, encontramos que E = 4.0286 1019 kJ. Como la entalpa de enlace del Li2(g) es de 102.8 kJ mol-1 (calculada en a)), necesitaramos 3.919 1017 mol de litio es decir, 5.44 1015 kg de litio! (si nos vemos pedantes, necesitaramos aun ms litio debido al equilibrio que hay entre el litio y el dilitio) e) Los clculos en d) indican que el dilitio no es un propulsor tan eficaz como han sugerido algunos investigadores. Sin embargo, el diberilio sera aun peor: la teora de orbitales moleculares muestra que el Li2(g) tiene un orden de enlace de 1, mientras que el Be2(g) no tiene carcter de enlace neto.

PROBLEMA 10
a) En una muestra de 100 g: 40.02 g son de C: esto equivale a (40.02 / 12.011) = 3.332 mol de tomos de C. 6.75 g son de H: equivalen a (6.75 / 1.00797) = 6.697 mol de tomos de H. 53.23 g son de O: son equivalentes a (53.23 / 15.9994) = 3.327 mol de tomos de O. La relacin C:H:O es de 1.001 : 2.013 : 1, por lo que la frmula emprica es CH2O. b) Tf = Kf M, donde Tf es la depresin del punto de congelacin y M es la molalidad de la solucin. El primer paso es determinar la molalidad de la solucin: Tf = 6.60 C - 2.02 C = + 4.58 C M = Tf / Kf, por lo que M = 0.229 mol kg-1. El siguiente paso es determinar la masa del disolvente: Masa de solucin = densidad vol = 0.777 g mL-1 500 mL 10-3 kg g-1 = 0.3885 kg. Masa de soluto = densidad vol = 10.44 g (o 0.01044 kg). Masa de disolvente = 0.3885 kg - 0.01044 kg = 0.3781 kg. La molalidad se define como el nmero de moles de soluto dividido entre la masa del disolvente (en kilogramos), por lo que: Moles(X) = 0.229 mol kg-1 0.3781 kg = 8.66 10-2 mol. La masa de X dividida entre el nmero de moles de X da la masa molar: Mr(X) = 10.44 g / 8.66 10-2 mol = 120.6 g mol-1. Ahora, considerando que la masa molar de la unidad formular CH 2O es de 30.03 g mol-1, obtenemos para X la frmula (CH2O)4.02, es decir C4H8O4.

Con respecto al uso del dilitio como combustible favorito en el siglo XXIV, lo nico que podemos asumir es que la tecnologa replicadora del siglo XXIV ha avanzado lo suficiente como para producir grandes cantidades de litio a partir de la nada

c)

La solucin a este problema es muy similar a la de b); la diferencia principal es que, en un disolvente polar como el agua, puede esperarse que haya ionizacin. Esta posibilidad (de disociacin en iones) nos obliga a usar la frmula Tf = i Kf M, donde el factor i indica el nmero promedio de fragmentos producidos por la disociacin de cada molcula de soluto. Tf = 0.00 1C - 3.54 C = + 3.54 C i M = Tf / Kf = 3.54 C / (1.86 C kg mol-1) = 1.903 mol kg-1. Masa de disolvente = (500 mL 1.005 g mL-1) - (50 mL 1.044 g mL-1) = 450.3 g. Usando la Mr(X) obtenida en b), determinamos que 50 mL de X equivalen a 0.433 mol. M = 0.433 mol / 0.4503 kg = 0.961 mol kg-1. Podemos entonces determinar el factor de disociacin i: i = 1.903 / 0.961 = 1.980. Entonces, parece ser que la disociacin de X en dos fragmentos es prcticamente cuantitativa en solucin acuosa. O, si asumiramos que i = 1, la masa molar que obtendramos para X por este mtodo sera de 120.6 g mol-1 / 1.980 = 60.9 g mol-1.

d)

Primero, calculemos el nmero de moles de hidrxido consumidos por esta reaccin: n(OH-) = 1.247 mol L-1 33.60 10-3 L = 4.190 10-2 mol. Como la reaccin es monitoreada por pH, podemos asumir que se trata de una titulacin cidobase. Entonces, el nmero de moles de OH - gastados reaccionaron con una cantidad equivalente de H+, produciendo agua. (Este argumento es necesario para poder determinar la masa del solvente.) Masa de solucin = vol densidad = 58.50 mL 1.003 g mL-1 = 58.68 g. Masa de soluto = (masa de Yi-) + (masa de Na+). Masa de Yi- = masa(X) - masa(H+) moles(OH-) = (25.00 mL 1.044 g mL-1 0.1) - 1.00797 g mol-1 0.0419 mol = 2.57 g. Masa de Na+ = 22.990 g mol-1 0.0419 mol = 0.96 g. Por lo que la masa de disolvente = 58.68 g - (2.57g + 0.96 g) = 55.15 g. Ahora, usando i M = Tf / Kf, obtenemos i M = 2.78 C / 1.86 C kg mol-1 = 1.495 mol kg-1. El parmetro i M representa el nmero de moles de especies disueltas por kilogramo de agua. Estas especies son Na+ y Yi-. Multiplicando por la masa del disolvente obtenemos n(Na+) + n(Yi-) = 1.495 mol kg-1 5.515 10-2 kg = 8.24 10-2 mol. Ya hemos determinado que n(Na+) es igual al nmero de moles de OH - consumidos, por lo que

0.1 = factor de dilucin (50 mL de X diluidos a 500 mL en c))

n(Yi-) = 8.24 10-2 mol - 4.19 10-2 mol = 4.05 10-2 mol. Como la relacin molar (Yi-) : (Na+) = 1 : 1.03, encontramos a partir del anlisis del punto de congelacin que la sal formada tiene la frmula Na +Y-. Dividiendo la masa de Yi- (encontrada anteriormente) entre los moles de Yi-, obtenemos que Mr(Yi-) = 64.4 g mol-1, lo cual es comparable con Mr(C2H3O2-), que es de 59.0 g mol-1. Podemos concluir que la sal formada es Na+C2H3O2-. Ntese, que si como se sugiere en b), la frmula de X es C4H8O4, debera haber (25.00 mL 1.044 g mL-1 / 120.6 g mol-1) = 2.16 10-2 mol de X. Entonces, la conversin de X en 2 C2H3O2- es prcticamente cuantitativa. e) Los resultados de b) y c) parecen estar en desacuerdo: en ciclohexano se encontr una masa molar dos veces mayor que la encontrada en agua. Esto podra resolverse si X se disociara completamente en dos iones en solucin acuosa: quiz X es un cido fuerte. Sin embargo, los cidos fuertes son relativamente raros en Qumica Orgnica. Adems, no esperaramos que un cido fuerte se disolviera fcilmente en un disolvente no polar como el ciclohexano. El resultado de d) representa un problema adicional: los dos productos de disociacin de X en solucin acuosa, se convierten aparentemente en C2H3O2-. Cmo sera esto posible, si los productos de disociacin fueran iones con cargas opuestas? Una explicacin alternativa, y que describe mejor lo que ocurre qumicamente, es que la forma disociada de X en H2O vista en c) es en realidad una especie sin carga, capaz de reaccionar como cido (como se vio en d)). La masa molar calculada de esta especie disociada neutra es de 60.9 g mol-1, lo que nos da la frmula C2H4O2. Existen pocos ismeros con frmula C2H4O2, y el que es ms probable que reaccione como cido es el cido actico (cido etanoico), de frmula CH3COOH. Entonces por qu X da, aparentemente, dos molculas de cido actico en solucin acuosa? La mejor respuesta consiste en considerar que X no se disocia en solucin acuosa: ms bien se dimeriza en ciclohexano. El dmero de cido actico, formado por puentes de hidrgeno
O CH3 C O H O H O C CH3

es simtrico y por lo tanto no polar y soluble en un disolvente no polar como el ciclohexano.

PROBLEMA 11
a) El cambio de energa libre se calcula usando G = H - TS. H = Hf(H) + Hf(D) - Hf(H2) -Hf(D) = -3.43 kJ mol-1. S = S(H) + S(HD) - S(H2) - S(D) = +4.48 J mol-1 K-1. Por lo que, a 20 K, G = -3.52 kJ mol-1, y a 1000 K G = -7.91 kJ mol-1.

La reaccin es espontnea en esta direccin para ambas temperaturas. (De hecho, como H es negativa y S es positiva, la reaccin debe ser espontnea a cualquier temperatura.) b) H es negativa en la direccin indicada; esto implica que la fuerza del enlace en HD es mayor a la de H2. Esto est de acuerdo con la tendencia general mencionada en el enunciado del problema, donde se seal que los istopos ligeros forman enlaces ligeramente ms dbiles. S es positiva en esta direccin, lo cual indica que los productos estn menos ordenados que los reactivos. Este aumento en el desorden puede explicarse considerando que hay dos maneras idnticas de formar la molcula HD a partir de una sola molcula H2. c) La conversin de HD a D2 est asociada a un cambio de entalpa negativo, ya que la fuerza del enlace en D2 es mayor que en HD (los istopos ms pesados forman enlaces ms fuertes). El cambio en la entropa tambin ser negativo, ya que (por argumentos similares a los usados en b)) el HD est ms desordenado que el D 2. Si suponemos que las magnitudes de H y S son iguales a las calculadas en a) (es decir, H = -3.43 kJ mol-1 y S = -4.48 J mol-1 K-1), obtenemos: G = -3.34 kJ mol-1 a 20 K, y G = +1.05 kJ mol-1 a 1000 K. Por consiguiente, la reaccin es espontnea a 20 K, pero a 1000 K la reaccin espontnea es la inversa. d) i) Como el equilibrio (1) siempre se encuentra hacia la derecha independientemente de la temperatura, es evidente que el D atmico no puede ser la forma ms abundante de deuterio. La intuicin indica que el HD predominar sobre el D2 si la abundancia total de deuterio es baja, pero podemos hacer un anlisis ms riguroso: Si se invierte el equilibrio (2), al combinarlo con (1) obtenemos H2 + D + D2 + H 2HD + H + D

o, cancelando los trminos que se encuentran en ambos lados, H2 + D2 2HD (3)

El clculo del cambio de energa libre para este equilibrio es G(3) = G(1) G(2); segn los clculos realizados en a) y en c), el valor es G(3) = -0.18 kJ mol-1. Como G(3) es negativo, el equilibrio debe estar desplazado hacia la derecha. Por lo tanto, el HD predominar sobre el D2. ii) El argumento anterior tambin se aplica si n(D) = n(h), por lo que el HD predominar sobre el D2. Adems, como el equilibrio (3) est desplazado hacia la derecha, HD tambin ser la forma predominante de hidrgeno.

PROBLEMA 12
a) El helio neutro tiene dos electrones; una especie hidrogenoide slo tiene un electrn. Por lo tanto, la especie en cuestin es He+.

b)

El espectro del tomo de hidrgeno obedece la relacin

1 1 E = R H 2 2 nf ni
por lo que, en el caso que estamos estudiando, las lneas del espectro del He+ deben obedecer la siguiente relacin
1 1 E = R He+ 2 2 nf 4

lo cual, considerando que E = h = hc / , puede rearreglarse para dar


R He + 1 1 = ( hc / ) 2 2 nf 4
1

Ahora debemos intentar ajustar el espectro a esta relacin. Si suponemos que la longitud de onda ms grande observada, 6558 (la cual es debida a la transicin menos energtica) corresponde a nf = 5, obtenemos lo siguiente: 6.558 10-7 5.410 10-7 4.858 10-7 4.540 10-7 4.338 10-7 nf 5 6 7 8 9 " RHe+" 1.35 10-17 J 1.06 10-17 J 0.97 10-17 J 0.93 10-17 J 0.91 10-17 J

Si la asignacin fuera correcta, todas las transiciones deberan dar el mismo valor de RHe+. Evidentemente, en este caso no fue as. Debemos intentar de nuevo. Si probamos con nf = 6 para la transicin de 6558 , obtenemos: 6.558 10-7 5.410 10-7 4.858 10-7 4.540 10-7 4.338 10-7 nf 6 7 8 9 10 " RHe+" 8.72 10-18 J 8.72 10-18 J 8.72 10-18 J 8.72 10-18 J 8.72 10-18 J

Esta es la asignacin correcta, ya que el valor de RHe+ obtenido es constante. c) d) EI(He+) = RHe+. Para convertir esto a electrn volts, multiplica por 6.02205 1023 mol1

y divide entre 96486 J mol-1 eV-1: EI(He+) = 54.44 eV.

EI(He+) / EI(He) = 2.180, por lo que EI(He) = 24.97 eV. Entonces EA(He2+) = 79.41 eV = 1.272 10-17 J. Podemos calcular la frecuencia, = E/h = 1.920 1016 s-1, y la longitud de onda = c/ = 15.61 nm, del protn menos energtico que es capaz de la doble ionizacin. Esta es evidentemente una longitud de onda mucho menor que la de la luz visible (entre 300 nm y 700 nm). El sol no es un cuerpo negro lo suficientemente caliente

como para producir estos fotones en abundancia, y de cualquier manera la mayora de ellos seran absorbidos por la atmsfera antes de llegar a la superficie terrestre.

PROBLEMA 13
a) Bsicamente, lo que la estudiante est intentando determinar es la masa molar [Mr(MX)] de cada muestra, intercambiando a M + por H+ en la columna de intercambio inico y determinando la cantidad de H+ liberado por medio de la titulacin. Las reacciones involucradas son las siguientes: M+ + [RSO3H] H+ + OHb) Anlisis de resultados: Moles de M+ en 5 g = moles de OH - (250 mL / 50 mL) (100 mL / 40 mL) = volumen de titulante 0.326 mol L-1 5 2.5. Mr(MX) = masa de la muestra (5 g) / moles de M+ en 5 g. Esto da como resultado lo siguiente: Muestra A B C D E F KBr Mr / g mol-1 58.01 41.88 165.81 57.88 119.13 42.09 119.71 MX posibles NaCl(58.44), KF(58.10) LiCl(42.39), NaF(41.99) KI(166.00), RbBr(165.37), CsCl(168.36) NaCl(58.44), KF(58.10) KBr(119.00), RbCl(120.92) LiCl(42.39), NaF(41.99) KBr(119.00), RbCl(120.92) H+ + [RSO3-M+] H2O

Debe apreciarse un inconveniente en los resultados anteriores: parece ser que algn aspecto de la tcnica experimental no es lo suficientemente preciso como para identificar las sales con certeza. Por ejemplo, si todas las muestras son en verdad diferentes, y la muestra etiquetada KBr es realmente KBr, entonces la muestra E debe ser RbCl. La masa molar del RbCl es significativamente mayor que la del KBr, pero la muestra E tiene la Mr menor de acuerdo con los resultados de la titulacin. Ntese que la gran similitud entre la masa molar de dos (o hasta tres) halogenuros alcalinos imposibilita la identificacin completa de A, B, C, D, o F. c) Vidrios de reloj: Las sales de litio son muy higroscpicas, por lo que si se dejan, por ejemplo, muestras de B y F en dos vidrios de reloj debera ser posible la identificacin del cloruro de litio: el cloruro de litio formar un charco en poco tiempo, siempre y cuando la atmsfera del laboratorio no est demasiado seca. Papel indicador: El ion fluoruro es la base conjugada de un cido dbil (HF), por los que los fluoruros resultan alcalinos en solucin acuosa. Por lo tanto, con el papel indicador debe ser posible identificar si B o F es NaF, y si A o D es KF.

Persulfato acidificado + almidn: El persulfato oxida al I - a I2, el cual forma un complejo de color azul oscuro con la solucin de almidn. Entonces, si C es KI, la adicin de persulfato + almidn dar un complejo de color azul profundo. El persulfato tambin oxida al Cl- y al Br-, pero estos no reaccionan con el almidn dando la coloracin caracterstica, por lo que este reactivo no permite distinguir entre RbBr y CsCl. Sin embargo, como la estudiante logr identificar a C usando las tcnicas descritas, necesariamente C tuvo que ser KI. No se realiz ninguna prueba para identificar el NaCl. Por eliminacin, aquel de A o D que no resulte alcalino debe ser NaCl. d) La propiedad que impide la identificacin segura de los halogenuros alcalinos con base en el intercambio inico es el hecho de algunos tienen casi la misma masa molar que otros. Esto se debe a que los compuestos son univalentes, y a que los metales alcalinos tienen un nmero atmico que es mayor al del halgeno anterior solamente en dos unidades; entonces, al quitarle una capa llena a un metal (p. ej. K Na) y agregarle una capa llena a un halgeno (p. ej. F Cl), obtenemos un compuesto con una masa molar casi idntica a la del compuesto original. En general, los halogenuros de los metales alcalinotrreos no tendrn este problema, siempre y cuando ambos tomos de halgeno sean iguales: al quitarle una capa llena al metal (p. ej. Ca Mg) y agregarle una capa llena al halgeno (p. ej. F Cl) obtenemos da masas molares bastante diferentes (p. ej. 78.08 g mol-1 para el CaF2 y 95.21 g mol-1 para el MgCl2).

PROBLEMA 14
a) b) c) d) El cubano parece tener seis anillos, correspondientes a las seis caras de un cubo. m = 2 (n +1 - r - d). Se puede comprobar esto examinando algunos ejemplos sencillos (tales como metano, etano, etino, benceno, etc.). Para n = 8, r = 6, d = 0, se obtiene un valor de m = 6 (es decir, C8H6). Sin embargo, la frmula del cubano es sin duda alguna C8H8. Coroneno: n = 24, r = 7, d = 12, lo cual da m = 12 (es decir, C24H12). Coranuleno: n = 20, r = 6, d = 10, lo cual da m = 10 (es decir, C20H10). Estos valores concuerdan con las estructuras esperadas:
H H H H H H H H H H H

H H H H H

H H H H

e)

El coranuleno (C20H10) no es plano. Si se consideran como rayos a los enlaces que surgen del pentgono central, entonces el ngulo preferido entre dos rayos es de 60

(como lo requiere la geometra de un hexgono regular). Sin embargo, un anillo formado por seis hexgonos regulares

slo puede cubrir un ngulo de 300. Cinco hexgonos son insuficientes para completar el anillo en un espacio plano. Al unir los extremos marcados con a, se logra cerrar el esqueleto del coranuleno, pero al hacer esto la molcula toma una forma de tazn. f) Cubano aplanado:
H C C H

C H H C H H

C H

C H

El cubano, al ser dibujado de esta manera, parece tener cinco anillos. Usando r = 5 obtenemos C8H8, lo cual concuerda con b) pero no con a). g) En el coroneno es posible romper siete enlaces carbonocarbono; en el coranuleno solamente seis. El nmero de enlaces que pueden ser rotos sin fragmentar la molcula es igual al nmero de anillos.
1 2 7 4 6 5 5 4 3 6 1 2 3

h)

En el cubano es posible romper cinco enlaces. Por lo tanto, el cubano tiene cinco anillos, lo cual est de acuerdo con b) y f), pero no con a). 1 2

5 4 3

i)

El dodecaedrano tiene 11 anillos. Esto nos da la frmula qumica correcta, C 20H20 al usar la frmula m = 2 (n +1 - r - d).
H C H C CH CH HC CH CH CH CH H C

CH HC C H

CH

CH C H CH

CH CH

C H

j)

Sesenta tomos de carbono, cada uno con tres vecinos inmediatos, nos da un total de 90 enlaces CC (60 3 nos da el doble de este nmero, ya que cada enlace estara contado dos veces). Cada tomo de carbono debe tener cuatro enlaces para satisfacer sus valencias, por lo que cada carbono debe participar en un doble enlace. Entonces d = 30 (recuerda que cada enlace tiene dos extremos). De la frmula del compuesto sabemos que n = 60 (tomos de carbono) y m = 0 (tomos de hidrgeno), por lo que r (el nmero de anillos) debe ser 31. Si 12 de los anillos son pentgonos, debe haber 19 hexgonos. Sin embargo, por analoga con las otras estructuras de jaula (el cubano y el dodecaedrano), o inspeccionando un baln de ftbol, puede verse que el C60 tiene 32 caras (desde un punto de vista geomtrico), y que 20 de esas caras son hexgonos. Una de las caras en la superficie del C60 no corresponde a un anillo verdadero, pero el asignar un pentgono o un hexgono como el anillo ilusorio es algo totalmente arbitrario. No podemos decir sin ambigedad cuntos anillos pentagonales y cuntos anillos hexagonales hay.

PROBLEMA 15
a) CO2 (g) CO2 (aq) + H2O HCO3- (aq) CO2 (aq) HCO3- (aq) + H+ (aq) CO32- (aq) + H+ (aq) (1) (2) (3)

Toma en cuenta que puede introducirse el equilibrio adicional CO2 (aq) + H2O H2CO3

para considerar que pueden existir tanto el CO 2 disuelto como el cido carbnico acuoso, pero este equilibrio no es estrictamente necesario para explicar qumica de carbonatos en agua. Como se establecer un equilibrio entre el lado izquierdo y el derecho de cada reaccin, y como empezamos con CO2 (g) y H2O, evidentemente la solucin resultante tiene que ser cida.

b)

El NH3 es un gas bsico: NH3 (g) NH3 (aq) + H2O NH3 (aq) NH4+ + OH- (aq)

por lo que se establecer una reaccin cidobase, desplazando los equilibrios (2) y (3) hacia la derecha. Esto har que aumente la tendencia del CO 2 atmosfrico a disolverse. El Cl2 es un gas cido: Cl2 (g) Cl2 (aq) + H2O HOCl (aq) Cl2 (aq) H+ (aq) + Cl- (aq) + HOCl (aq) H+ (aq) + OCl- (aq)

El aumento en [H+] generado por estas reacciones desplazar los equilibrios (2) y (3) hacia la izquierda. Esto har que disminuya la tendencia del CO 2 atmosfrico a disolverse. Por lo tanto, la tendencia del CO2 a disolverse sigue este orden: mezcla ii) > mezcla i) > mezcla iii). c) El acetato, CH3COO-, es la base conjugada de un cido dbil: CH3COO- (aq) + H2O CH3COOH (aq) + OH- (aq)

La solucin de acetato de sodio es considerablemente alcalina, por lo que desplazar los equilibrios (2) y (3) hacia la derecha. La solucin de cido clorhdrico desplazar estos mismos equilibrios hacia la izquierda. Por lo tanto, la tendencia del CO2 a disolverse sigue este orden: solucin iii) > solucin i) > solucin ii). d) La concentracin de CO2 acuoso est dada por la ley de Henry: [CO2 (aq)] = KH P(CO2) = 3.39 10-2 mol L-1 atm-1 3.5 10-4 atm = 1.187 10-5 mol L-1. Es conveniente convertir los valores de Kb a Ka, usando Ka = Kw / Kb. Ka(CO2 (aq)) = 4.46 10-7. Ka(HCO3- (aq)) = 4.67 10-11. Estos valores de Ka se definen como [HCO3 ] [H +] Ka(CO2 (aq)) = 4.46 10-7 = [CO2 (aq)] Ka(HCO3
(aq)

) = 4.67 10

-11

[CO32] [H +] = [HCO3 ]

Como Ka(CO2 (aq)) Ka(HCO3- (aq)), podemos asumir que en una solucin cida slo la primera desprotonacin ser significativa (podemos comprobar esto, una vez que hayamos calculado [H+]). Por lo tanto, [H+] = [HCO3-] = (4.46 10-7 1.187 10-5) = 2.30 10-6 mol L-1. Es decir, pH = 5.64. Ahora, usando [H+] = [HCO3-] = 2.30 10-6 mol L-1, podemos ver que [CO32-] = 4.67 10-11 mol L-1. Por lo tanto, el grado de disociacin de HCO 3- en H+ y CO32- es muy pequeo, y nuestra suposicin (de que la segunda desprotonacin es insignificante) result correcta. e) Debe resultar evidente por intuicin que 1 atm de CO 2 considerablemente ms cida que 350 ppm de CO2 los equilibrios CO2 (g) CO2 (aq) + H2O CO2 (aq) HCO3- (aq) + H+ (aq)
(g) (g)

producir una solucin

. Entonces, por las mismas

razones presentadas en d), para resolver este problema slo necesitamos considerar

[CO2 (aq)] = KH P(CO2) = 3.39 10-2 mol L-1, y [H+] = [HCO3-] = (Ka [CO2 (aq)]) = 1.23 10-4 mol L-1. Es decir que pH = 3.91. f) Esta es una reaccin de titulacin de un cido dbil diprtico con una base fuerte. Primero necesitamos determinar [CO2 (aq)]total: [CO2 (aq)] total = [CO2 (aq)] + [HCO3-] + [CO32-] = (1.187 10-5) + (2.30 10-6) + (4.67 10-11) mol L-1 = 1.417 10-5 mol L-1. por lo que la alcuota de 100 mL contiene 1.417 10-6 mol del cido dbil diprtico CO2
(aq)

y sus bases conjugadas. Esto requerir 2.834 10-6 mol de OH- para

neutralizarse, o sea 28.34 mL de NaOH 1.00 10-4 mol L-1. Al inicio de la titulacin, el pH es de 5.64, como vimos en d). En 7.1 mL (a la mitad del primer punto de equivalencia), pH = pKa(CO2 (aq)) = 6.35. En 21.2 mL (el punto medio entre el primero y el segundo puntos de equivalencia), pH = pKa(HCO3-) = 10.33.

PROBLEMA 16
a) Existen cuatro ismeros estructurales: CH3 CH3CH2 C* C OH H O cido 2-metilbutanoico CH3 CH3 C C OH O O H3C

CH3CH2CH2CH2 C OH O cido pentanoico CH3 CH3 C CH2 C OH H

cido 3-metilbutanoico

cido 2,2-dimetilpropanoico

El cido 2-metilbutanoico es pticamente activo. b) La acidez involucra la disociacin para formar un anin carboxilato: O R C O H R C O O + H+

La posicin del equilibrio depende de la estabilidad del anin RCOO-. Como los grupos alquilo son donadores inductivos de densidad electrnica, las estructuras ms ramificadas tendrn los valores de Ka ms bajos. El orden esperado de Ka (De mayor a menor) es el siguiente: cido pentanoico cido 3-metilbutanoico cido 2-metilbutanoico cido 2,2-dimetilpropanoico (no tiene ramificacin en la cadena) (metilo en el carbono ; desestabilizado ligeramente por efecto inductivo) (metilo en el carbono ; ms desestabilizado que el anterior) (dos metilos en el carbono ; el ms desestabilizado de todos)

Podemos comparar esta prediccin con los valores de la literatura (para ver si el razonamiento usado es vlido). En el orden propuesto, los valores de pKa son 4.76, 4.76, (el del cido 2-metilbutanoico no est reportado), y 5.04, lo cual concuerda esencialmente con lo predicho tomando en cuenta el efecto inductivo. c) Hay 9 ismeros estructurales de este tipo (adems de algunos que no son pticamente activos):
H H C H H C H F C* H COOH H H C H F C* H H C H COOH

C H

C H

H H H F C* H H C H H C H COOH H C C H H F C* H COOH

C H

C H

H H H H C H C* F COOH C H H H C H H

F C C* H H COOH

C H

C H

H H H F C* H C* H COOH C H F H C H H

H C C* H H COOH

C H

C H

H H F C C* H H H C H COOH

C H

El flor atrae electrones de manera inductiva, por lo que la proximidad del tomo de flor al grupo carboxilo estabilizar la base conjugada, y por lo tanto aumentar la fuerza del cido. La especie ms cida ser A, porque tiene el tomo de flor en la posicin al carboxilo (lo cual le da la mxima estabilizacin por F) y no tiene cadenas alqulicas ramificadas (lo que le da la mnima desestabilizacin por grupos metilo). La menos cida de las especies pticamente activas es H, ya que tiene el flor en la posicin al cido carboxlico (mnima estabilizacin por atraccin de electrones por F) y adems tiene un grupo metilo en la posicin al grupo COOH (mxima donacin de electrones por ramificacin de la cadena alqulica). d) Esta reaccin produce un ster: RCOOH + ROH e) El ster marcado es RCOOR + H2O

H3 C

C O

O*

CH2 CH2 CH3

La reaccin consiste en un ataque nucleoflico al tomo de carbono del carboxilo:

R'

C O

OH

f)

i)

El producto esperado es

H3 C

C O*

CH2 CH2 CH3

ii)

Cuando se le agrega cido etanoico a la solucin de NaOH, se establece un equilibrio cidobase: CH3CO*OH + OH CH3(COO*)- + H2O

El ion etanoato tiene dos oxgenos equivalentes. Cuando se vuelve a acidificar la solucin, la etiqueta queda revuelta: CH3(COO ) + D
* +

CH 3C
18

OD

O
18

CH3C O

OD

con los dos isotopmeros formados en igual proporcin (siempre y cuando el sistema cido etanoico / hidrxido de sodio haya tenido tiempo de llegar al equilibrio). El primer isotopmero dar el mismo producto obtenido en i); el segundo dar etanoato de propilo no marcado y agua marcada (para ser precisos, H18OD).

PROBLEMA 17
a) El volumen de una esfera es V = (4/3) r3. r = 3.5 10-10 m, por lo que V = 1.796 10-28 m3. Ahora la presin, P = nRT / V. n = 1 tomo (6.02205 1023 mol-1)-1 = 1.661 10-24 mol. por lo que P = 2.2907 104 kPa (229 atm). b) Primero debemos calcular el nmero de moles de C60 en 1 gramo: Mr(C60) = 60 12.011 = 720.66 g mol-1, por lo que 1 g = 1.388 10-3 mol. Ahora debemos calcular el nmero de moles de 3He que se obtuvieron a partir de 1 g de C60. n = PV / RT (P = 1.00 105 Pa, T = 298 K)

por lo que n(3He) = 1.00 105 Pa 1.15 10-13 m3 / (8.314 J K-1 mol-1 298 K) n(3He) = 4.642 10-12 mol / g de C60. (o sea que solamente una de cada 2.99 108 molculas de C60 contiene un tomo de
3

He) Denotaremos la fraccin de molculas de C60 que contienen 3He como f(3He) = 3.34 10-9. Entonces la presin parcial media efectiva es: Pmedia(3He) = P(He@C60) f(3He) = 7.66 10-5 kPa. Si procedemos de manera similar para el 4He: n(4He) = 8.436 10-9 mol / g de C60, (f(4He) = 6.08 10-6), y Pmedia(4He) = 0.139 kPa. c) Patm = 1.00 102 kPa. La presin parcial de 4He en la atmsfera es igual a la presin total fraccin mol de
4

He:

P(4He)atm = 1.00 102 kPa 5.24 10-6 = 5.24 10-4 kPa. La presin parcial de 4He en la atmsfera terrestre es menor a la de la muestra por ms de dos rdenes de magnitud. La interaccin qumica entre el C 60 y el 4He es prcticamente nula, por lo que no puede favorecer la captura espontnea de helio por el C60. Resulta difcil de explicar cmo puede ser mayor la presin de 4He en la muestra que en la atmsfera. Par lo tanto, el contenido de 4He en la muestra no sustenta la hiptesis de que la muestra se gener en la atmsfera terrestre. (En el problema no se menciona la posibilidad de en esa poca la presin atmosfrica de la Tierra haya sido cientos de veces mayor a la actual. Es decir que para la misma fraccin mol de helio, s sera probable que la muestra se hubiera formado en la Tierra. Sin embargo, esta no es una situacin muy probable.) d) La relacin 3He/4He en la muestra es de 5.50 10-4:1, o sea que es mayor por dos rdenes de magnitud que la relacin en la atmsfera terrestre. (Entonces, la presin parcial de 3He en el fullereno de la muestra excede al valor actual en la atmsfera terrestre por un factor de 1.12 105!) La relacin de abundancia de los istopos definitivamente no concuerda con un origen terrestre del helio atrapado en el fullereno. Esto implica que el He@C60 ya exista en el meteorito, y sobrevivi al impacto. (Como dato curioso, actualmente se piensa que el He@C60 en el meteorito no solamente tiene un origen extraterrestre, sino tambin extrasolar. No hay ambientes en el sistema solar que tengan una presin de helio lo suficientemente alta y no tengan adems una presin alta de H2 o algn de otro gas que inhiba la formacin de fullerenos. Los mejores candidatos para el origen del He@C60 [ver Science 272 (1996), p. 249] son un tipo de estrellas gigantes rojas con alto contenido de helio y carbono y bajo contenido de hidrgeno, que se sabe que estn rodeadas por partculas de holln. Todas estas estrellas estn a cientos de aosluz de distancia, por lo menos.)

e)

i)

Para determinar el rea superficial del fullereno, tenemos que definir una celda unitaria bidimensional de grafito: 4.26

2.46

La celda unitaria tiene lados de longitud ( r(CC) [2 + 2 sen(30)]) y (r(CC) 2 cos[30]), respectivamente, por lo que el rea de la celda unitaria es de 1.084 10-19 m2. La celda unitaria contiene: 2 tomos de carbono completos (negros) 2 medios tomos de carbono (grises) 4 cuartos de tomo de carbono (blancos) En total, 4 tomos de carbono.

Por lo tanto hay un tomo de carbono por cada 2.62 10-20 m2, y como hay en total 5 109 tomos, el rea superficial total del fullereno es A = 1.31 10-10 m2. El rea de una esfera es A = 4 r2, por lo que el fullereno tiene un radio de 3.23 10-6 m. El volumen de una esfera es V = (4/3) r3, por lo que V = 1.409 10-16 m3. ii) El esqueleto de carbono del fullereno tiene una densidad C = masa / V: masa = 5.0 109 tomos 12.011 g mol-1 / (6.02205 1023 tomos mol-1). masa = 9.97 10-14 g. Entonces C = 708 g m-3. La atmsfera de helio en el interior del fullereno tiene la densidad:

He = (PHe / Patm) (Mr(He) / Vmol),


donde Vmol es el volumen molar a TPS (es decir, 2.2414 10-2 m3), por lo que He = 214 g m-3. La densidad total del fullereno es tot = C + He = 922 g m-3. iii) La densidad del aire es

aire = Mr(aire) / Vmol = 1294 g m-3.


iv) El fullereno lleno de helio es menos denso que el aire: un slido ms ligero que el aire! (O, si prefieres, puedes considerarlo como un globo molecular de helio.)

PROBLEMA 18
a) Si denotamos a la actividad inicial con I0 (7.0 107 Bq en todos los casos), y con It a la actividad una vez transcurrido un tiempo t, I t queda definida como It = I0 e-kt, donde k es la constante de decaimiento, y es igual a ln(2) / t1/2. Es conveniente convertir todos los tiempos de vida media a segundos, para que todas las constantes de decaimiento estn en unidades del SI: t1/2(71Zn) = 2.4 min 60 s min-1 = 144 s, k(71Zn) = ln(2) / t1/2 = 0.6931 / (144 s) = 4.814 10-3 s-1; t1/2(67Ga) = 78.25 h 60 min hr-1 60 s min-1 = 2.817 105 s, k(67Ga) = 0.6931 / (2.817 105 s),= 2.461 10-6 s-1; t1/2(68Ge) = 287 d 24 h d-1 60 h min-1 60 s min-1 = 2.480 107 s, k(68Ge) = 0.6931 / (2.480 107 s) = 2.795 10-8 s-1. Con esto ya podemos determinar It, e It despus de la dilucin (este ltimo valor es igual a It / 2500). t = 30 min 60 s min-1 = 1800 s. nclido Zn 67 Ga 68 Ge b)
71

It (Bq mL-1) 1.21 104 6.97 107 6.9996 107

It despus de la dilucin (Bq mL-1) 4.83 2.79 104 2.80 104

El 71Zn tiene una vida media demasiado corta como para permanecer activo por suficiente tiempo: despus de 30 minutos prcticamente toda la actividad ha cesado. El nmero de cuentas es muy bajo para ser confiable, especialmente despus de ser diluido en toda la sangre del paciente. Adems, una vida media tan corta implica que el nclido debe ser sintetizado para cada paciente: tiene un tiempo de almacenamiento despreciable. El 68Ge tiene el problema opuesto: es casi igual de activo despus de 30 minutos, y con una vida media de casi un ao, permanecer activo por mucho tiempo. Si el nclido es retenido en el cuerpo del paciente, esto implica que el paciente estar expuesto a una dosis de radiacin inaceptablemente alta durante todo este tiempo, con el dao correspondiente a sus clulas, etc. El 67Ga tiene un tiempo de vida media que es lo suficientemente largo para ser conveniente, pero lo suficientemente corto como para poder hacer una medicin confiable del volumen sanguneo usando una dosis relativamente pequea de material radiactivo, minimizando as los riesgos para la salud del paciente.

c)
71 30Zn 67 31Ga 68 32Ge

+ +
0 -1e 0 -1e

71 31Ga 67 30Zn 68 31Ga

0 -1e

(+ )

(+ ) (+ )

d)

i)

El producto radiofarmacutico contena inicialmente (1.025 10-2 g / 69.72 g mol-1) = 1.47 10-4 mol de Ga, y por lo tanto (1.47 10-4 5.0 10-7) = 7.35 10-11 mol de 67Ga (o 4.43 1013 tomos de este nclido). Para el decaimiento radiactivo, la cintica de primer orden es: velocidad = It = k nt(67Ga) (donde It est dado en bequerels y nt(67Ga) es el nmero de tomos de 67Ga presentes al tiempo t). Sustituyendo los datos que ya se tienen se obtiene It = 1.09 108 Bq. En la dosis de 1 mL cuando t = 8 h, It = I0 e-kt Vdosis / Vtotal = 1.09 108 Bq e-2.461 10-6 28800 1/100 It = 1.02 106 Bq.

ii)

La actividad residual de la dosis de 1 mL despus de una hora sera It = 1.02 106 Bq e-2.461 10-6 3600 = 1.01 106 Bq. Comparando esta actividad con la obtenida para la muestra de sangre de 1 mL, obtenemos el factor de dilucin: Factor de dilucin = 1.01 106 / 105.6 = 9564. Por lo tanto el volumen sanguneo del paciente es de 9.56 L.

PROBLEMA 19
a) El uranio metlico tiene un nmero de oxidacin de cero, por definicin. Los nmeros de oxidacin de las otras especies son: U(III) [U 3+]; U(IV) [U4+]; U(V) [UO2+]; U(VI) [UO22+]. Las condiciones descritas son las condiciones estndar, por lo que podemos usar directamente los potenciales estndar para determinar en que sentido son espontneas las reacciones. Tambin debemos considerar el paso de reduccin: 2H+ + e H2 E = 0.000 V por definicin

b)

Dos semirreacciones involucran al uranio metlico: U3+ + 3eUO22+ + 4H+ + 6e U U + 2H2O

stas sern el punto de partida para la oxidacin del uranio: 2U + 6H+ U + 2H+ 2H2O 2U3+ + 3H2 UO22+ + 3H2 Ecelda = +1.798 V Ecelda = +1.444 V

Independientemente de cul de estos procesos se vea ms favorecido, ninguno de los productos primarios, U3+ o UO22+ ser el producto final. El U(III) se oxida espontneamente a U(IV): 2U3+ + 2H+ 2U4+ + H2 Ecelda = +0.607 V

Y el U(VI) se reduce espontneamente a U(IV) o U(V):

UO22+ + 2H+ + H2 2UO22+ + H2

U4+ + 2H2O 2UO2+ + 2H+

Ecelda = +0.327 V Ecelda = +0.062 V

El U(V) tambin se reduce espontneamente a U(IV): 2UO2+ + 6H+ + H2 2U4+ + 4H2O Ecelda = +0.620 V

Como el U4+ es la nica especie que no reacciona espontneamente con H + o H2, esta es la especia que predominar finalmente en solucin acuosa bajo estas condiciones. [Puede tomarse en cuenta que, mientras haya uranio metlico presente, el proceso U4+ + 3U 4U3+ Ecelda = +1.191 V

es favorable, pero slo puede ocurrir hasta que se agote U, despus de lo cual el U(III) se oxidar como se vio anteriormente.] c) Como las condiciones son estndar excepto porque [H +] = 1.0 10-6 mol L-1, podemos usar una forma simplificada de la ecuacin de Nernst, como la que se muestra a continuacin, para las ecuaciones relevantes: 2U + 6H+ 2U3+ + 3H2 Ecelda = +1.798 V + -6 Ecelda = Ecelda - (RT / 6F) ln([H ] ) = +1.444 V U + 2H+ + 2H2O UO22+ + 3H2 Ecelda = +1.444 V + -2 Ecelda = Ecelda - (RT / 6F) ln([H ] ) = +1.326 V 2U3+ + 2H+ 2U4+ + H2 Ecelda = +0.607 V + -2 Ecelda = Ecelda - (RT / 2F) ln([H ] ) = +0.253 V UO22+ + 2H+ + H2 U4+ + 2H2O Ecelda = +0.327 V + -2 Ecelda = Ecelda - (RT / 2F) ln([H ] ) = -0.027 V Todos los procesos mostrados arriba tienen valores positivos de Ecelda para la reaccin en la direccin en que est escrita, por lo que todos son espontneos a pH = 0. Sin embargo, a pH = 6, U(VI) U(IV) y U(V) U(IV) son espontneos en la direccin inversa (como puede verse en los valores negativos de Ecelda). Ahora el U(IV) se oxida espontneamente a U(V) o a U(VI). Como ahora U(VI) es el nico estado de oxidacin que no reacciona espontneamente con H+ o con H2, la especie predominante en estas condiciones ser UO22+. d) Para resolver esto, debemos considerar nicamente las reacciones donde aparece UO2+: 2UO22+ + H2 2UO2+ + 2H+ Ecelda = +0.062 V + 2 Ecelda = Ecelda - (RT / 2F) ln([H ] P(H2)-2) 2UO2+ + 6H+ + H2 2U4+ + 4H2O Ecelda = +0.620 V + -6 Ecelda = Ecelda - (RT / 2F) ln([H ] P(H2)-2) En las condiciones lmite, Ecelda = 0, lo cual corresponde a los siguientes casos: [H+] = {P(H2)2 e(2FEcelda / RT)} para U(VI) U(V), y [H+] = {e(-2FEcelda / RT) / P(H2)2}1/6 para U(IV) U(V).

Estas expresiones dan valores lmite de: i) [H+] < 11.2 mol L-1, o pH > -1 para que U(V) sea estable con respecto a U(VI), [H+] < 3.19 10-4 mol L-1, es decir, pH > 3.50 para que el U(V) sea estable con respecto a U(IV). ii) [H+] < 1.12 10-5 mol L-1, o sea pH > 4.95 para que el U(V) sea estable con respecto al U(VI). [H+] < 3.19 10-2 mol L-1, es decir, pH > 1.50 para que el U(V) sea estable con respecto al U(IV). Entonces, el UO2+ parece ser ms estable que otros estados de oxidacin en un rango de pH de 3.57 (recuerda que solamente estamos considerando soluciones cidas y neutras) bajo una atmsfera de hidrgeno estndar. Si la presin es de 1.0 10-6 atm, el UO2+ solamente es estable con respecto a su oxidacin a U(VI) si el pH es mayor a 4.95. La presin parcial de H2 en la atmsfera es muy pequea, por lo que las condiciones dadas en ii) son las que se asemejan ms a las encontradas en un entorno terrestre. Sin embargo, el UO2+ no es en realidad tan estable como lo sugieren sus potenciales estndar de celda.

PROBLEMA 20
a) El color se debe al dixido de nitrgeno, NO 2. Como el aire tiene 78% de N2 y 21% de O2, el oxgeno es el reactivo limitante: si se convierte totalmente el O 2 a NO2 (lo cual es muy poco probable), la concentracin de dixido de nitrgeno ser: [NO2] = 0.21 (% en mol de O2) / 22.414 L mol-1 (vol. molar en TPS) = 9.4 10-3 mol L-1. b) c) 2NO + O2 2NO2 i) El orden con respecto a NO y O2 se puede obtener a partir de aquellas series de mediciones donde la concentracin de alguno de ellos se mantenga ms o menos constante (por ejemplo, [NO] se mantiene prcticamente constante en las mediciones #1, 2 y 3, mientras que [O2] se mantiene casi constante en #2, 4 y 5): Orden con respecto al NO: Mediciones comparadas #4 : #2 #4 : #5 #2 : #5 Relacin entre las concentraciones de NO 2.01 4.02 2.00 Relacin entre las velocidades iniciales 4.03 15.9 3.95

Puede observarse que la rapidez vara con [NO]2, por lo que la reaccin es de segundo orden con respecto a NO.

Orden con respecto a O2: Mediciones comparadas #2 : #1 #2 : #3 #1 : #3 Relacin entre las concentraciones de O2 1.99 3.85 1.93 Relacin entre las velocidades iniciales 1.98 3.65 1.84

La rapidez inicial vara directamente con [O2], por lo que la reaccin es de primer orden con respecto a O2, y globalmente la reaccin es de tercer orden. ii) La ecuacin de velocidad de la reaccin es la siguiente: Rapidez de reaccin = k [NO]2[O2], por lo que k = rapidez / ([NO]2[O2])

Las diversas mediciones dan los siguientes resultados: Medicin #1 #2 #3 #4 #5 k 7.063 103 L2 mol-2 s-1 7.154 103 L2 mol-2 s-1 7.159 103 L2 mol-2 s-1 7.117 103 L2 mol-2 s-1 7.165 103 L2 mol-2 s-1

El valor promedio es k = 7.13 103 L2 mol-2 s-1. d) i) Las mediciones #1, 2 y 3 tienen prcticamente la misma concentracin inicial de [NO], con variaciones considerables en [O2]: el exceso estequiomtrico de [O2] sobre [NO] es slo de 7.2 10-6 mol L-1 en la medicin #3, mientras que en la medicin #2 es de 3.67 10-4 mol L-1. Entonces, si la reaccin no se llevara a cabo completamente, la relacin A:[NO]inicial debera variar considerablemente. Sin embargo, para las tres primeras mediciones, la relacin A :[NO]inicial es casi constante. Esto implica que la reaccin se ha llevado a cabo de manera prcticamente completa. ii) La ley de Beer define la absorbancia: A = log(I0/I) = l C, donde C = [NO2] = [NO]inicial, ya que la reaccin se ha llevado a cabo completamente y NO fue el reactivo limitante; l = 10 cm. Si usamos la medicin #1, por ejemplo, obtenemos = A / C l = 0.341 / (1.16 10-4 10) = 294 L cm-1 mol-1. iii) El NO2 es caf rojizo. De los colores del espectro visible, el ms parecido a ste es el naranja; entonces, si se est transmitiendo el naranja, es porque se est absorbiendo el color complementario (el azul). Esto concuerda con un mximo de absorcin alrededor de 400 nm.

e)

Comparemos el volumen con PV: V (mL) 1000 500 200 100 50 20 10 PV (atm mL) 2.49 2.45 2.36 2.25 2.12 1.92 1.78

PV es proporcional al nmero de moles de gas presente en el recipiente. Evidentemente, el nmero de moles est decreciendo al disminuir el volumen. Esto se debe al siguiente equilibrio: 2NO2 N2O4 (g) Si asumimos que la presin a V = 1000 mL se debe nicamente al NO2 (lo cual es una aproximacin inicial razonable, ya que PV cambia muy poco entre 1000 mL y 200 mL), podemos determinar P(NO2) y P(N2O4) para cualquier otro volumen, considerando que hay una conversin de 2 mol 1 mol al llevarse a cabo la reaccin de izquierda a derecha. P(N2O4) = P(1000 mL) (1000 / V) - Ptot P(NO2) = Ptot - P(N2O4) Por ejemplo, para la medicin de 10 mL, P(N2O4) = 7.1 10-2 atm P(NO2) = 0.107 atm KP = P(N2O4)/(P(NO2)2) = 6.20. Ahora podemos usar este valor para probar si era correcta nuestra hiptesis de que solamente se encuentra NO2 cuando V = 1000 mL. Como P(N2O4) = 6.20 P(NO2)2, y suponiendo que P(N2O4) P(NO2), encontramos que P(N2O4) = 3.84 10-5 atm (y restando de Ptot, P(NO2) = 2.45 10-3 atm) para V = 1000 mL. Esto implica que debemos corregir nuestras ecuaciones para las presiones parciales: P(N2O4) = 2.53 10-3 atm (1000 / V) - Ptot P(NO2) = Ptot - P(N2O4) donde 2.53 10-3 atm es igual a 2P(N2O4) + P(NO2) cuando V = 1000 mL, es decir, la presin parcial que habra si todo el N 2O4 se convirtiera en NO2, en un volumen de 1000 mL. Ahora podemos volver a calcular KP a partir de la medicin con V = 10 mL. [Cualquier otra medicin servira, pero el volumen ms pequeo tendr la relacin P(N2O4):P(NO2) ms grande y, por lo tanto, la incertidumbre mnima en el valor de KP.] Con V = 10 mL, obtenemos

P(N2O4) = 7.5 10-2 atm, P(NO2) = 0.103 atm, KP = P(N2O4)/(P(NO2)2) = 7.07. Usando este nuevo valor de KP con los datos de V = 1000 mL, obtenemos P(N 2O4) = 4.24 10-5 atm. Aunque este valor es un poco mayor que el obtenido en la primera iteracin, no altera significativamente la cantidad 2P(N 2O4) + P(NO2). Hemos obtenido una solucin consistente dentro de la precisin de los datos: KP = 7.07. f) Esto se debe a la condensacin del N2O4 a una presin lo suficientemente alta: N2O4 (g) N2O4 (l) La presin se estabiliza debido a que el vapor de N2O4 est en equilibrio con la forma lquida, por lo que P(N2O4) no puede exceder su presin de vapor; adems, sigue existiendo el equilibrio entre N2O4 y NO2, regido por el valor de KP. Como la presin de N2O4 est fija, tambin la de NO2 tiene que mantenerse fija. Por lo tanto la presin permanecer constante aunque se comprima ms el sistema (por lo menos hasta que empiece a comprimirse el lquido). Podemos determinar P(N2O4) y P(NO2) a partir de KP y Ptot: 2NO2 (g) N2O4 (g) P(NO2) = x atm P(N2O4) = (1.215 - x) atm KP = 7.07 = (1.215 - x) / x2 7.07x2 + x - 1.215 = 0.

Al resolver esta ecuacin cuadrtica encontramos que P(NO2) = x = 0.350 atm y P(N2O4) = 0.865 atm, usando los datos con V = 1 mL. La constante de equilibrio para la condensacin del N 2O4 es, por definicin, KP = P(N2O4)-1. (La actividad del N2O4 (l) puro es igual a la unidad, por lo que no aparece en la constante de equilibrio.) Como P(N2O4) = 0.865 atm cuando est en equilibrio con el lquido, encontramos que KP = 1.156.

PROBLEMA 21
a) Se obtienen los siguientes valores: pH 2 6 10 Y43.712 10-14 2.249 10-6 0.3548 [Y4-] / mol L-1 8.26 10-16 5.01 10-8 7.90 10-3

Las concentraciones de los aniones se calculan usando CT(EDTA) = 0.02226 mol L-1 (Mr(C10H16N2O8) = 292.25 g mol-1).

b)

Debemos determinar la relacin [MY2-] / [M2+], que (considerando la definicin de la constante de complejacin) es igual a KY [Y4-]. Podemos determinar [Y4-] a partir de CT(EDTA) y las Y4- determinadas en a). Esto nos da los siguientes valores: pH 2 6 10 [Y4] 1.856 10-16 1.1245 10-8 1.774 10-3 [HgY2-] / [Hg2+] 1.17 106 7.08 1013 1.12 1019 [FeY2-] / [Fe2+] 0.039 2.36 106 3.73 1011 [CaY2-] / [Ca2+] 9.28 10-6 562 8.87 107

Puede verse que a pH = 2, solamente el Hg2+ forma un complejo de manera prcticamente cuantitativa; a pH = 6, lo forman tanto Hg 2+ como Fe2+; mientras que a pH = 10 los tres metales forman complejos con alto rendimiento. c) Como el HCl es un cido fuerte, el equilibrio entre Hg 2+ y Cl- debe ser independiente del pH: podemos calcular que [HgCl42-] / [Hg2+] tendr un valor de 2.488 1014 si [Cl-] = 0.5 mol L-1. Usando los valores de [HgY2-] / [Hg2+] calculados en b), obtenemos lo siguiente: pH 2 6 10 d) [HgY2-] / [Hg2+] 1.17 106 7.08 1013 1.12 1019 %(Hg2+) 4 10-13 3 10-13 9 10-18 %(HgY2-) 5 10-7 22.2 > 99.9 %(HgCl42-) > 99.9 78.8 2.2 10-3

Los resultados de b) indican que a pH = 2 la complejacin de Ca 2+ con EDTA es despreciable. Podemos suponer que esto tambin es cierto para pH = 2.6, por lo que el EDTA solamente est reaccionando con el Hg 2+. A pH = 10, la complejacin tanto de Hg2+ como de Ca2+ es cuantitativa, por lo que a pH = 9.5 la titulacin nos permite conocer la cantidad total de Hg2+ y Ca2+. Debemos suponer que el EDTA no reacciona de manera significativa con Na+. A pH = 2.6, moles de Y4- gastados = 5.391 10-4 mol. Esto es igual al nmero de moles de Hg2+ en 25 mL, por lo que los moles de Hg2+ en 500 mL sern 1.078 10-4 mol. Como la masa atmica del Hg es de 200.59 g mol -1, esto representa una masa de mercurio de 2.163 g en la muestra de amalgama. A pH = 9.5, moles de Y4- gastados = 7.006 10-4 mol. Esto es igual a moles de (Hg 2+ + Ca2+) en 10 mL, por lo que moles de (Hg 2+ + Ca2+) en 500 mL = 3.503 10-2 mol; por lo tanto en 500 mL hay 2.425 10-2 mol de Ca2+. Esto corresponde a una masa de calcio de 0.972 g. Por diferencia, la masa de sodio en la muestra (suponiendo que no hay ningn otro componente) es de 2.083 g. Entonces, la composicin de la muestra es: Hg (41.45%), Na (39.92%) y Ca(18.63%).

PROBLEMA 22
a) Los gases de invernadero calientan la parte baja de la atmsfera, debido a que algunos fotones infrarrojos (originados en la superficie de la Tierra), que normalmente

escaparan de la atmsfera, son absorbidos y reemitidos; la reemisin tiene tanta probabilidad de ser hacia la superficie terrestre como hacia afuera, por lo que resulta en un calentamiento de la atmsfera baja. b) Los gases de invernadero enfran las capas altas de la atmsfera: como menos fotones infrarrojos emitidos en la superficie llegan a esta altitud, la absorcin de radiacin infrarroja a 15 km resulta menor de lo normal. Menor absorcin implica menor temperatura. Los equilibrios en cuestin son: CO2 (aq) CO2 yH2O (l)
(g)

c)

H2O (g)

Cuando aumenta la temperatura, ambos equilibrios se desplazan hacia la derecha, por lo que la concentracin de vapor de agua y CO 2 aumentar con la temperatura. Como ambas especies son gases de invernadero, hay cierto grado de retroalimentacin positiva. d) En una atmsfera de puro oxgeno, la produccin fotoqumica del ozono debe involucrar la fotlisis del oxgeno: O2 + h seguida por O + O2 O3 (ii). O + O (i),

La fotlisis tambin destruye el ozono: O3 + h O2 + O (iii).

[Otro proceso posible para la destruccin del ozono es el siguiente: O3 + O e) O2 + O2 (iv).]

A partir de las entalpas de formacin dadas (y la del O 2, que por definicin es igual a cero), obtenemos las entalpas de reaccin para (i) y (iii): H(i) = 498 kJ mol-1 = 8.27 10-19 J molcula-1 H(iii) = 106 kJ mol-1 = 1.76 10-19 J molcula-1. Ahora, usando E = h = hc/ (h = 6.626 10-34 J s; c = 2.79979 108 m s-1), obtenemos: (i) = 2.40 10-7 m = 240 nm, (iii) = 1.129 10-6 m = 1129 nm. Estos son los fotones de mayor longitud de onda que tienen la energa suficiente para fotolizar al O2 y al O3, respectivamente. (En realidad, la fotlisis eficiente del ozono requiere de fotones con longitud de onda menor a la calculada, debido a que la formacin de productos en el estado basal est prohibida por simetra; pero esto est fuera del objetivo de este problema.)

f)

Los tres equilibrios involucran la formacin de un enlace al ir de izquierda a derecha, por lo que el equilibrio se desplazar ms hacia la izquierda a altas temperaturas y hacia la derecha a bajas temperaturas. Es posible rechazar tres de las hiptesis: i) La idea de que los niveles de CFCs en el hemisferio norte estn atrasados con respecto a los del hemisferio sur resulta contradictoria, ya que la mayor parte de las industrias y del uso de CFCs se encuentran en el hemisferio norte. (En realidad, la concentracin de CFCs en la parte baja de la atmsfera es relativamente homognea en todo el mundo: los CFCs han tenido tiempo de dispersarse bastante bien.) Aunque es probable que un aumento en la concentracin de vapor de agua aumente la probabilidad de formacin de NPE, en ninguna parte la informacin dada anteriormente indica que est ocurriendo un aumento en la cantidad de vapor de agua que llega a la estratosfera rtica (solamente a la atmsfera baja). El efecto de aumentar las concentraciones de gases de invernadero (que por cierto, todava estn aumentando) es la disminucin en la cantidad de radiacin IR que llaga a la estratosfera. Adems, aunque los fotones del infrarrojo cercano tienen ms energa que la del enlace O2O, realmente no destruyen el ozono. La hiptesis ii) tiene la respuesta ms razonable los gases de invernadero calentarn la atmsfera baja a costa de enfriar la estratosfera. (Esto no implica necesariamente que este mecanismo sea la causa verdadera los sistemas qumicos naturales tienden a ser ms complejos de lo que esperamos!)

g)

iii)

iv)

PROBLEMA 23
Como C = 65.2% H = 8.75% O = 26.05% Esto corresponde a la frmula emprica C10H16O3. La masa molar relativa de esta molcula sera (10 12.01 + 16 1.008 + 3 16.00) = 184.13 Como nos dicen que la masa molecular de Q est por debajo de 200, resulta que la frmula emprica de Q es igual a su frmula molecular. C10H16O3 Luego el problema nos dice que Q es cido, por lo que podemos suponer que tiene grupos COOH. De ser as, solamente puede haber un grupo carboxilo ya que hay en total 3 oxgenos y cada grupo requiere 2. Esto nos hace pensar que Q tiene que ser un cido monoprtico. Esto implica que 1 mol de Q reaccionara con 1 mol de NaOH: RCOOH + NaOH RCOO-Na+ + H2O Ahora, 43.7 mg
43.7 g de Q reaccionarn con 23.7 mL de NaOH 0.0100 M. 1000

Esto equivale a Por lo que, si

23.7 0.0100 mol de NaOH. 1000

43.7 23.7 g de Q reaccionan con 0.0100 mol de NaOH, 1000 1000

43.7 1000 g de Q reaccionarn con 1 mol de NaOH. 1000 23.7 0.01

Es decir, 184.3 g de Q reaccionan con 1 mol de NaOH. Esto demuestra que 1 mol de Q reacciona con 1 mol de NaOH y, por lo tanto, Q es un cido monoprtico del tipo RCOOH. Ahora debemos decidir a qu corresponde el tercer oxgeno en el compuesto C 10H16O3 (o C9H15OCOOH). Este podra ser: un ter un alcohol una cetona R C o un aldehdo H Todava no es posible decidir entre estas cuatro posibilidades. Para continuar debemos calcular el nmero de instauraciones (#ins) de Q. Recuerda que, para CaHbOc, En el caso de Q, #ins = 3. De stas, todava tenemos que explicar dos (2), pero una (1) ya la conocemos, porque
C O

ROR ROH R C O R O

#ins =

(2a + 2) b 2

est en el grupo

OH

. H+ /

Ahora examinemos la qumica del problema: Q H2 /Pt A NaBH 4 /EtOH B C + H 2O

C es un alqueno, lo cual sugiere que B es un alcohol, porque pierde agua al ser calentado con H+. Adems, como C presenta un grupo metilo unido a un doble enlace, podemos suponer que la tercera reaccin es las siguiente: CH3 CH CH OH R CH3 C H C H R

Si esto es correcto, A debe ser una cetona. Entonces la secuencia sera: NaBH4 CH3 C CHR CH3 CHCHR CH3 CH O (A) OH (B) (C)

CHR

(Se asume que sabes que los aldehdos y cetonas son reducidos por el NaBH 4 para dar alcoholes primarios y secundarios, respectivamente.) Si A contiene una cetona adems del cido carboxlico, podemos considerarlo as: CH 3 C O con lo que han sido explicadas 2 de las 3 insaturaciones. Una en el cido y una en la cetona. Esto implica que queda por explicar una insaturacin, y como Q reacciona con el hidrgeno, la insaturacin restante debe ser un doble enlace. (Se asume que sabes que es posible hidrogenar los dobles enlaces carbonocarbono.) El problema restante consiste en saber en que posicin se encuentra el doble enlace. Esto se resuelve en la parte final del problema. Ya sospechamos lo siguiente:
CH3 C(CH2 )7 CO2 H O (A) (B) (C) CH3 CH(CH2 )7 CO2 H CH3 CH CH(CH2 )6 CO2 H

(CH 2 )7

CO 2 H

Nuestra hiptesis es confirmada por el hecho de que C se rompe con el ozono y un oxidante para dar cido actico y un cido dicarboxlico de cadena lineal. CH 3 CH CH(CH 2 )6 CO 2 H CH 3 C O HO O C(CH 2 )6 CO 2 H HO Con esto queda definido completamente C, por lo que ya estamos seguros de la estructura de A. Pero la ozonlisis y oxidacin de Q nos da HO C C OH O O adems de E. En realidad no necesitamos preocuparnos por E, ya que el fragmento pequeo (de cido oxlico) solamente puede provenir de una molcula del tipo R CH CH CO2 H R C O + O C C O OH Con esto, Q queda definido como CH 3 C O El nico problema que queda es el de la isomera cis/trans del doble enlace. (CH 2 )5 CH CH CO 2 H OH OH

CH3 O

CH3 C (CH2)5 C H cis C CO 2H H o O

C (CH2)5 C H trans C CO2H H

Con la informacin disponible no es posible decidir cul de estos ismeros es Q. Puntos importantes a estudiar: Grado de insaturacin (tambin llamado equivalentes de doble enlace). Grupos funcionales. Reduccin. Ozonlisis con tratamiento posterior oxidativo o reductivo, especialmente de molculas que contengan varios grupos funcionales (no solamente dobles enlaces).

PROBLEMA 24
La solucin completa al problema es la siguiente:
OH O

OCH3

OCH3

OCH3 O

OH

OCH3 D CH3

CH3 B

CH3 C

CH3 O

F OH O OCH3 OH O OCH3 OH OH OCH3

CH3 O O E

CH3 OH

CH3

OH

OH

CH3 O

cido crisofnico

Notas acerca de la resolucin del problema: a) b) El compuesto B es el producto esperado de la reduccin de Birch. Como C es un ismero conjugado de B, puede tener cualquiera de las siguientes estructuras:

OCH 3

OCH 3

OCH 3

CH 3 C-1 C-2

CH 3 C-3

CH 3

De estas estructuras, solamente C-1 y C-2 concuerdan con los datos de RMN. Por lo tanto se descarta la estructura C-3. c) Cualquier producto de adicin de Diels-Alder obtenido a partir de C-1, C-2 o C-3, sera aceptable como respuesta a esta pregunta, ya que todos ellos tienen un grupo hidroxilo que forma un puente de hidrgeno intramolecular. Aunque esto es relativamente fcil de responder, la respuesta nica al problema requiere una mayor reflexin. En esta parte del problema todava no es posible distinguir cul es la estructura correcta para C (C-1 o C-2). Al final, sin embargo, resulta claro que las respuestas generadas usando D-1 o D-11, que son los productos de cicloadicin de C-1, no son aceptables, ya que estos productos no sern desmetilados con el BCl 3. Este reactivo, por lo menos a -10 C, es selectivo y solamente rompe los teres metlicos en posicin peri a un carbonilo; en este sentido es ms selectivo que el BBr3. La solucin completa, que se muestra arriba, es la obtenida usando el ismero correcto (C-2).
OH O OCH 3 OH O CH 3

OCH 3 O D-1 CH 3 O D-11

Sin embargo, aun as existe la posibilidad de otro producto, pero no es el que da finalmente el cido crisofnico. La regioqumica de la cicloadicin mostrada en la solucin del problema es la que ocurre en la prctica, pero la reaccin de Diels-Alder podra haber dado tambin el producto que se muestra abajo. Esto no hubiera dado como producto el cido crisofnico, pero como en el problema se da la estructura, es posible deducir la respuesta. Vale la pena sealar que la sntesis no est libre de ambigedad, por lo que no podra usarse como prueba de la estructura del cido crisofnico. Con esta informacin es posible responder la pregunta d), que trata sobre la regioqumica de la cicloadicin de C-2 y la 5-hidroxinaftalen-1,4-diona.
HO O CH 3 HO O CH 3

OCH 3

OH

Puntos importantes a estudiar: Reduccin de Birch de compuestos aromticos. No se pide mecanismo. Reaccin de Diels-Alder. Regioqumica, o direccin de la adicin.

Concepto de RMN. No se piden detalles sobre el acoplamiento, sino solamente una idea de la informacin que pueden dar al qumico los datos de RMN sencillos. Enolizacin de cetonas. Oxidacin de fenoles a quinonas. El BCl3 como agente desmetilante selectivo, que solamente desmetila los teres que estn en posicin peri al carbonilo. (Esto es porque se necesitan dos oxgenos el del carbonilo y el del grupo metoxi para formar un quelato con el boro.) Estrategias para resolver problemas de sntesis en varios pasos.

PROBLEMA 25
Me 3SiH 2CH 2CO 2C Me 3SiH 2CH 2CO 2C Me 3SiH 2CH 2CO 2C CO 2Me C

O O B

O OH B

Me 3SiH 2CH 2CO 2C CHO F

Me 3SiH 2CH 2CO 2C CH 2OH D

Me 3SiH 2CH 2CO 2C

Me 3SiH 2CH 2CO 2C

O H

CHO

O J

Me 3SiH 2CH 2CO 2C

CHO O I

HO 2C

O K

Notas acerca de la resolucin del problema: A simple vista el problema parece muy difcil. Sin embargo el problema es rico en informacin y pone a prueba la comprensin de las reacciones, junto con el conocimiento de algunas oxidaciones, reducciones y esterificaciones sencillas. La parte ms difcil del problema es la preparacin para la conversin de B a C.

Me3SiCH2CH2OCO O H O: CH3 CH3O C OCH3 OCH3 H+

Me3SiCH2CH2OCO O
+

CH3

CH3O OCH3

Me3SiCH2CH2OCO O O OCH3 Intermediario para el rearreglo de Claisen

Me3SiCH2CH2OCO O O CH3
+

H OCH3

O H

No es necesario conocer con tanta profundidad el mecanismo ya que en la introduccin del problema se da una pista importante acerca de como debe funcionar la qumica en trminos generales. La oxidacin con clorocromato de piridinio es notable, ya que bajo condiciones anhidras el alcohol primario se oxida solamente al aldehdo y no al cido carboxlico. Los reductores no tienen ningn significado especial en este problema y no es necesario conocerlos como reactivos especiales. Simplemente logran reducciones selectivas, pero la naturaleza de stas est esbozada en el problema. La conversin de C a D debe involucrar la reduccin del ster metlico, ya que D todava contiene silicio. Si se hubiese reducido el ster de trimetilsililetilo, se habra perdido el silicio. Este es un reto interesante para el estudiante, y no requiere un conocimiento previo de la reactividad de los distintos tipos de steres. Simplemente se deduce a partir de la composicin elemental. Finalmente, vale la pena notar que en esto problema se usa el smbolo para un reactivo de Wittig, donde se muestra en la estructura un doble enlace formal y no la estructura dipolar comnmente usada. Es importante observar que el reactivo G efectivamente nos permite extender la longitud de la cadena de un aldehdo. Puntos importantes a estudiar: Rearreglos de tipo Claisen. Otro ejemplo que podran analizar los estudiantes es el rearreglo de los teres allicos de los fenoles. Oxidacin y reduccin. Esterificacin con alcoholes simples y complejos. Reaccin de Wittig. Isomera y notacin (E) / (Z). Grado de insaturacin.

PROBLEMA 26
REDUCCIONES (i)

OCH 3 (ii)

CO2 H (iii)

(iv)

(v) CO 2 Me

(vi)

CO2Me

(Nota: un exceso de reductor puede reducir al alqueno polarizado)

(vii)

(viii)

OH

OXIDACIONES (ix)

CO 2 H

CO 2 H (x)

CO 2 H

(xi) CHO

(xii)

H O O H

(xiii)

O CO2

(xiv)

(xv) OH (xvi) HO HO HH OTRAS REACCIONES (xvii) O O O No hay reaccin excepto a temperaturas extremas

(xviii) (xix)

(xx)

CO 2 Me

(xxi) CO 2 Me El estudiante debe estar consciente de que el producto de la reaccin de Wittig puede ser cis o trans dependiendo de las condiciones de reaccin y las estructuras de los reactivos.

(xxii)

O CO 2 Me CO 2 Me

(xxiii)

H O O H O O O C CH3

(xxiv)

(xxv) O O (xxvi) CH 3

(xxvii)

OH OH

OH (xxviii) O O CH3 O

ndice Alfabtico
A
absorbancia acetileno cido etilendiaminotetraactico cidos carboxlicos afinidad electrnica agua carbonatada de cristalizacin de lluvia agujero de ozono alcohol almidn amalgama amiduro de potasio amortiguador de pH anlisis elemental anillos aromaticidad atmsfera autoionizacin azul de Turnbull 19 1 21 16 4 16 37 16 22 16 13, 32 21, 35 24 21 10, 23 13 5 17, 22 8 35 19, 20 1, 5 25 25 6 21 1 18 22 8 1 19 17 25 19 21, 34 8, 17 28 12 10 21 9, 20 20 12 19 36 29 1, 13 16 18 10 23 5 10 24 33 8 10, 20 15, 21 dixido de nitrgeno dodecaedrano 19 1, 15 21, 29 22 10 5 8 31 16 9 12 4, 12 11 3, 12 10 4, 9, 10 4 1, 2 24 8, 11, 22 8 11 15 20 19 22 18 8 18 11 7, 24 12 11 30, 36 7 25 16, 25 22 1 6 12, 30 35 10 22 12, 22 22, 34 34 7 12 7, 15, 17 17 21 4, 17 15 4 6 4, 11, 17

E
EDTA efecto invernadero efecto isotpico electrones Electrospray Mass Spectrometry eluyente enantimero energa cintica de aparicin de ionizacin libre de Gibbs niveles de vibracional enlace fuerza de orden de qumico enolizacin entalpa de formacin Enterprise entropa equilibrio cido-base colateral fisicoqumico lquido-vapor redox ESMS especiacin espectro de absorcin espectrometra de masas espectroscopa atmica espontaneidad estandarizacin estereoisomera estereoqumica esterificacin estratosfera etino

B
base conjugada benceno borohidruro de litio borohidruro de sodio bromo buffer butatrieno

C
captura electrnica CFC ciclo termoqumico ciclobutadieno cintica qumica clatratos clorocromato de piridinio coeficiente molar de absorcin complejo composicin isotpica compuestos de coordinacin configuracin electrnica constante crioscpica de disociacin de equilibrio de formacin de Rydberg de velocidad de reaccin copia heliogrfica cromatografa de intercambio inico cubano curva de valoracin

F
fenantreno fenolftalena ferricianuro de potasio frmula emprica fotlisis fotn fotoqumica fotosensible fragmentacin frecuencia fullereno

D
decaimiento radiactivo densidad deshidratacin de alcoholes deslocalizacin deuterio Diels-Alder, adicin de digestin dilitio dimerizacin dixido de carbono

G
gases de invernadero nobles solubilidad geometra grafito

H
helio

hibridacin hidrocarburo hidrocarburos aromticos policclicos hidrogenacin cataltica hidrogenoide

1, 7 13 5, 14 23 12 33 25 12, 29 34 8 24 7 2, 6 28 7 16 7, 16 8, 10, 16 8 28 4 20 20

I
ignicin iluro intercambio inico intramolecular, reaccin redox ion ion molecular ionizacin isomera de enlace estereoismera estructural ptica istopo isotopmero IUPAC

precipitacin presin de vapor parcial puente de hidrgeno punto de congelacin,depresin del punto de ebullicin punto final

32 9, 20 17 24 10 7 10, 37 10 7, 16 23 18 18 18 7 24 29, 35 23, 26 4 7 12, 30 6 23, 24 24 18 5 24 23, 24 24 11 8, 11 23 32 10, 12 21, 29 36 24 6 18 18 18 25 4 30

Q
qumica de bajas temperaturas quiralidad

R
radiacin IR radiactividad radioistopos radionclidos rearreglo rearreglo de Claisen recristalizacin reduccin reductor relacin masacarga resina de intercambio inico resonancia resonancia magntica nuclear

K
kriptn

L
ley de Lambert-Beer ligante longitud de enlace de onda

S
sal de Fremy semirreaccin simetra singuletes sntesis orgnica

4, 6 11, 19, 22, 34 7 16 8 16 18 17 5 30 24 10 22 8 8 4, 18

M
m/z marcado, tomo masa molar mecanismo medicina nuclear meteorito

T
tautomera ceto-enol teora cuntica termoqumica tetrahidrofurano tiosulfato de sodio titulacin cido-base complejomtrica redox tricloruro de boro trifenileno

N
naftaleno naranja de xilenol nitrodisulfonato de potasio nube interestelar nube polar estratosfrica nclido nmero de masa nmero de oxidacin

U
uranio

O
orbital antienlazante atmico molecular orden de reaccin oxalato de sodio oxidacin oxidante oxgeno ozono ozonlisis 2 2 2, 4 19 36 23, 24, 26 4 22 22 23 5 18 5 36 13 16, 20 10 6 24

V
vida media volumen sanguneo

W
Wittig, reactivo de

X
xenn

Z
zinc constante de acidez

P
PAHs partcula periodicidad permanganato de potasio persulfato de sodio pH pHmetro pireno pirlisis

16

Vous aimerez peut-être aussi